PHYSICS FORM FOUR REGIONAL EXAMS (MOCK, PRE-NATIONAL, etc.)

PRESIDENT’S OFFICE REGIONAL ADMINISTRATION AND LOCAL GOVERNMENT 

LINDI - REGION 

FORM FOUR PRE NATIONAL EXAMINATION

031 PHYSICS

Time: 3 Hours September 2023

Instructions

  1. This paper consists of section A, B and C with a total of eleven (11) questions.
  2. Answer all questions in section A and B and two (2) questions from section C.
  3. Section A carries fifteen (15) marks, section B sixty (60) marks and section C carries twenty five (25) marks
  4. Non programmable calculators and mathematical tables may be used.
  5. Write your examination number on every page of your answer sheet.
  6. Where necessary the following constants may be used.
  • Acceleration due to gravity, g=10 m/s2 or 10 N/kg
  • Density of water =1.0 g/cmPie,=3.14
  • Speed of sound is 340 m/s

SECTION A (16 Marks)

Answer all questions in this section

1. For each of the items (i)-(x), choose the correct answer from among given alternatives, and write its letter beside the item number in the answer booklet provided. (10 marks)

i. A stone is thrown horizontally from the top of a building at a speed of 12 m/s. how long does it take to fall 45 meters vertically? (Assume negligible air resistance)

  1. 1.0 sec 
  2. 5.0 sec 
  3. 3.0 sec 
  4. 8.0 sec 
  5. 2.14 sec

ii. In a race competition, one should kneel down when starting the race so that

  1. No one can fall down by reaction produced by the earth to them
  2. Each one can receive an acceleration which the earth provides
  3. One of the rules in sports is not violated by anyone
  4. Equal and opposite forces can act on a body to produce a resultant force.
  5. Each one could exert a force on the surface of the ground

iii. Heat would be lost in the thermos flask if the walls of the glass container were not coated with silver. Which process contributes to this kind of heat loss?

  1. Radiation
  2. Conduction
  3. Convection
  4. Absorption
  5. Transmission

iv. In a light experiment, observing an object through a certain material showed that less light was transmitted and the image was distorted. Which type of material was used

  1. A translucent material
  2. An opaque material
  3. A luminous material
  4. A transparent material
  5. A non-luminous material

v. Why oil is used as lubricant

  1. It has low density
  2. It is highly viscous
  3. It is flammable
  4. It is less viscous
  5. It is costle

vi. When silicon element is doped with phosphorus atom, it produces:

  1. P-type semiconductor
  2. Force
  3. intrinsic semiconductor
  4. bipolar semiconductor
  5. n-type semiconductor

vii. A body is said to be in equilibrium if

  1. It moves with uniform speed
  2. The net force acting on it is zero
  3. The upward and downward forces are equal
  4. Its center of gravity is low positioned
  5. Its center of gravity is high

viii. What happens when a liquid changes into gaseous state?

  1. Some surface molecules absorb latent heat of vaporization and escape
  2. It gives its own latent heat that can be used to heat up the surrounding
  3. The potential and kinetic energies of the molecules increase
  4. The molecules attractive forces to one another increases and their average kinetic energy decreases
  5. There is no adhesive force between molecules.

ix. What is the main function of step up transformer?

  1. To change a.c to d.c current
  2. To decrease resistance in a circuit
  3. To increase a.c voltage
  4. To decrease a.c voltage
  5. To increase a.c current

x. Which of the following arguments describes the mathematical language used in physics?

  1. Density is mass per volume
  2. Matter occupies space
  3. Volume is the amount of space occupied by the body
  4. Physics is the branch of science
  5. Action and reaction are equal and opposite

2. Match the items in List A with response in List B by writing the letter of the correct response beside the item number in the answer booklet provided. (05 marks)

List A

List B

  1. Longitudinal wave
  2. Reverberation
  3. Snell’s law
  4. Critical angle
  5. Principle of superposition
  6. Electromagnetic waves
  1. Does not need material medium
  2. Refractive index
  3. Water waves
  4. Sound wave
  5. Angle of incidence for which the angle of refractive is 90°
  6. Rise and fall of sounds of two vibrating objects
  7. The resulting displacement at any point is equal to the sum of displacements of different waves
  8. The multiple reflection of sound waves
  9. Angle of reflection for which the angle of incidence is 90°
  10. The resulting of the wave length

SECTION B (54 Marks) 

Answer all questions in this section

3. (a) Form one students from Mzumbe secondary school visited Kilimanjaro Mountain. When climbing the Mountain to high altitude, one of the students got the problem of nose bleeding.

  1. Comment on why student got such problem? ( 02 marks)
  2. Why astronauts wear space suits? (02 marks)

(b) The acceleration due to gravity on Jupiter is about 2.6 times that on the earth. A spacecraft has a weight of 24500 N on earth.

(i) What is the mass of the spacecraft? (03 marks)

(ii) What would be its weight on Jupiter? (03 marks)

4. (a) What is the refractive index for a certain medium, if the light in air enters the medium at an angle of 30° and refracted at 22°? ( 04 marks)

(b) A vertical object 10 cm high is placed 20 cm away from a convex mirror of radius of curvature 30 cm. using a ray diagram, determine (06 marks)

(i) Image distance

(ii) The height of the image formed

(iii) The magnification of the image

5. (a) (i) What do you understand by the term Specific heat capacity and Specific latent heat of vaporization ( 02 marks)

(ii) Explain the factors which affect the boiling point of water. (03 marks)

(b) A sample contains 800 g of Iodine-131. How much Iodine-131 in the sample will remain undecayed after 40 days, If the Half-life of Iodine-131 is 8 days? (05 marks)

6. (a) Mr. Joearmstrong put the coin on a card placed over the mouth of a bottle. When the card is flicked away with the finger the coin drops neatly into the bottle.

(i) Which law demonstrated by Mr.Joearmstrong? (02 mark)

(ii) State the law identified in (i) above (02 marks)

(b) A screw jack has a screw pitch of 5mm and the effort arm 16 cm.

( i) State two forms of energy in which the energy supplied to the screw jack is finally converted to ( 02 marks)

(ii) Determine the percentage efficiency of this screw jack, if it needs an effort of 30N to lift a load of 750N. (04 marks)

7. (a) Describe how a lens camera operates the same as human eye. Give three points (06 marks)

(b) Briefly explain how conduction of heat can be applied in your daily life (three reasons) (04 marks)

8. Weather forecasting shows that there is the possibility of occurring earthquake in Mtakuja village. The villagers are not aware with such disaster. As physicist, make villagers aware with

  1. Meaning of earthquakes ( 02 marks)
  2. The four key indicators observed before an earthquake occurs (04 marks)
  3. Suggest four precautions to be taken by villagers during an earthquake in order to minimize injuries and deaths. (04 marks

SECTION C (30 MARKS)

Answer two (02) questions in this section.

9. (a) George Ohm observed that as the current flows through the circuit, it encounters some opposition. This opposition determines the amount of current flowing in electric device depending to the particular material.

(i) State the law that Mr. George formulate. (02 marks)

(ii) Briefly explain factors affecting resistance of a conductor observed by Mr. George Ohm to sum up his observation. (04 marks)

(b) (i) Distinguish between the concept of conductors, semiconductor and insulators in term of energy bands (04.5 marks)

(ii) Give out one structural difference between A.C and D.C generators. (02 marks)

10. (a) Briefly explain how length of the string affects frequency of vibrating string. (02 marks)

(b) A string has length of 75 cm and a mass of 8.2 g. the tension in the string is 18 N. What are the frequencies of the 1st and 3rd harmonics? (06 marks)

(c) A transformer is used to step down 240 V mains supply to 12 V for laboratory use. If the primary coil has 600 turns, determine the number of turns in the secondary coil. (04.5 marks)

11. (a) Why some semiconductors called P type and other called N type? (3 marks)

(b) Describe, how you will connect the semiconductor diode as forward bias. (4 marks).

(b) The jaws of a vernier caliper touch the inner wall of calorimeter without pressure. The position of zero of vernier scale on the main scale reads 3.4 cm. the 6th of vernier scale division coincides with the main scale division. Vernier constant of calipers is 0.01 cm. Find actual internal diameter of calorimeter, when it is observed that the vernier scale has a zero error 0.03cm. (5.5 marks)

FORM FOUR PHYSICS EXAM SERIES 185  

FORM FOUR PHYSICS EXAM SERIES 185  

THE UNITED REPUBLIC OF TANZANIA PRESIDENT’S OFFICE REGIONAL ADMINISTRATION AND LOCAL GOVERNMENT

 ZONAL FORM FOUR MOCK EXAMINATIONS 

(NJOMBE, MBEYA, SONGWE, RUKWA AND KATAVI REGIONS)

031/1 PHYSICS 1

TIME 3:00 HOURS YEAR: 2023

INSTRUCTIONS

  1. This paper consists of sections A, B and C with total of eleven (11) questions.
  2. Answer all questions in sections A and B and only two(2) questions from section C.
  3. Section A carries sixteen (16) marks, section B fifty four (54) marks and section C carries thirty (30) marks.
  4. Cellular phones and any unauthorized materials are not allowed in the examination room.
  5. Non-programmable calculators and mathematical tables may be used.
  6. Write your Examination Number on every page of your answer booklet(s).
  7. Where necessary the following constants may be used.

(i) Acceleration due to gravity, g = 10m/s2.

(ii) Density of water = 1.0g/cm3.

(iii) Speed of light = 3x108 m/s

(iv) Specific heat capacity of water = 4200 J/kg0C

SECTION A:

Answer all questions in this Section.

1. For each of the items (i) – (x), choose the most correct answer from among of the given alternatives and write its letter beside the item number in the answer sheet provided. (10 Marks)

(i) A bus carrying heavy load on its top carrier is likely to overturn because;

  1. It runs faster 
  2. Its center of gravity is low 
  3. Its center of gravity is high
  4. Its equilibrium is neutral 
  5. It is at stable equilibrium

(ii) The extremely narrow bore on the liquid-in-glass thermometer in which the thermometric liquid flows during expansion or contraction is known as;-

  1. Capillary tube 
  2. Steel index 
  3. Stem 
  4. Constriction 
  5. Bore

(iii) Which of the following conditions must be satisfied for a body to float?

  1. Apparent weight is equal to the difference between real weight of the body and itsupthrust
  2. Upthrust equal to the weight of the fluid displaced
  3. Real weight of the body equals to its upthrust
  4. Apparent weight is equal to the product of real weight of the body and its upthrust
  5. Density of a body is equal to the density of surrounding fluid

(iv) The capacitance across MJ in the diagram below is

  1. 11 µF 
  2. 0.09 µF 
  3. 1.05 µF 
  4. 1.59 µF 
  5. 10.5 µF

(v) Radio set rated 400W is switched on for 10 hours every day. How much energy does it consume in 15 days?

  1. 21600KJ 
  2. 216000KJ 
  3. 216000000KJ 
  4. 2160000KJ 
  5. 216KJ

(vi) In a cloud chamber, straight-line trait (streaks) of a vapor are produced by a sources emitting . . . . . . . . . . . . .

  1. Beta-particle  
  2. Gamma – rays 
  3. Alpha – particles 
  4. Light rays. 
  5. UV- light

(vii) A sheet of white paper is viewed through a piece of blue glass and the paper looks blue.

  1. The colour of the glass is reflected on to the paper 
  2. Blue light is absorbed by the glass
  3. Blue light travel faster than red through glass 
  4. The glass absorb all colors except blue
  5. Blue is unique colour.

(viii) In an experiment, A simple pendulum swings between A and B. The amplitude of oscillation is

  1. Distance A to B 
  2. Half the distance A to B 
  3. Distance A to B and back
  4. Twice the distance A to B 
  5. The distance from A in one direction

(ix) A spiral spring of length 1.50 m naturally, is extended to 1.505 m by a force of 0.80 N. What will be its extension when the applied force is 3.2 N?

  1. 0.005m 
  2. 6.020m 
  3. 1.57m 
  4. 4.520m 
  5. 0.020m

(x) If an elevator is going up with acceleration, apparent weight of the body is

  1. More or less than the true weight 
  2. Equal to the true weight
  3. Less than the true weight 
  4. More than the true weight
  5. Less than apparent weight

2. Match the items in list A with the responses in list B by writing the letter of the correct response beside the item number. (06 Marks)

LIST A

LIST B

image

  1. Returning spring
  2. Hydraulic fluid
  3. Sliding pedal
  4. Master cylinder
  5. Brake drum
  6. Brake shoes
  7. Slave cylinder
  8. Brake pedal
  9. Spring balance

SECTION B: (54 Marks)

Answer all questions in this Section

3. (a) A body dipped in a liquid experiences an Upthrust. Explain three (03) factors on which the upthrust depends.(04 Marks)

(b) Juma is an optician, he tested the eye sight and noticed that his client can only see division of scale 1.5 mm in size at near point. He wishes to prescribe the spectacles such that each division appear 9 mm when read through. Calculate the focal length of the lens he will fit into the frame to make such spectacles. (05 Marks)

4. (a) Heat generated in the car engine has to be removed, effectively using a liquid in the car cooling system to avoid overheating. Table I below shows the characteristics of some liquids used in the cooling system.

Table 1.

Liquid

Characteristics

Specific heat capacity (Jkg0c-1)

Freezing point (0c)

Boiling point (0c)

Rusting rate on metal

J

5000

20

110

High

K

4600

-15

120

Low

L

3800

15

95

Medium

M

3000

5

95

Medium

N

200 -

20

320

Low

With reference to table 1, which liquid is most suitable to be used in the cooling system? Justify your choice. (05 Marks)

(b) A screw jack has a screw pitch of 5mm and the effort arm of 16cm

(i) State two forms of energy in which the energy supplied to the screw jack is finally converted to.

(ii) Determine the percentage efficiency of this screw jack, if it needs an effort of 30N to lift a load of 750N. (04 Marks)

5. (a) (i) Engineer Kanon was structuring a door at the finishing, he fixed the knob near the hinge. Was Engineer Kanon correct? Briefly explain by giving reason to your answer. (02 Marks)

(ii) A metal bench feels colder than a wooden one when one sits on it on a cold morning even though are at the same temperature. Explain this observation. (02 Marks)

(b) Form four physics teacher entered in the class with hydrometer marked 25 OC. As a form four student assist your friend who is form one to explain why is a definite temperature marked on commercial form of hydrometer ( 05 Marks)

6. (a) Rutherford proton-neutron model shows that there are no electrons in the nucleus of an atom. How is it possible for a radioactive element to emit beta particles (electrons) from its nucleus? (04 Marks)

(b) Two identical free running trolleys are on a smooth horizontal runway. One trolley is at rest and the other approaches it at constant speed of 20 m/s.

(i) Using the principle of conservation of momentum, find the common speed of two Trolleys after the collision.

(ii) Why the kinetic energies before and after the collision are different? (05 Marks)

7. (a) A bottle containing ammonia solution is placed at the back of the laboratory. Give a reason why its smell may not be detected in other parts of the laboratory if the temperature of the solution is kept very low. (04 Marks)

(b) (i) What is a windmill?

(ii) Mention three disadvantages of energy caused by wind.

(iii) Does wind itself possess energy? Explain. (05 Marks)

8. (a) A pressure cooker will cook beans faster than an open saucepan. Give explanation on this observation. (04 Marks)

(b) An alloy of copper and tin has a volume of 1005cm3. The density of copper is 8.905g/ cm3 and of tin 7.305g/cm3. How much volume of each metal must be used if the alloy is to have the density of 7.625g/cm3? (05 Marks)

SECTION C: (30 Marks)

Answer only two (2) questions from this Section.

9. (a) Briefly explain why a piano can be distinguished from a violin even when they are playing notes of the same pitch. (03 Marks)

(b) During a storm, thunder is heard 7s after the lightning is seen. If the temperature of the air at the time of the storm is 25 OC, How far away is the storm cloud? (05 Marks)

(c) A string A is 2m long and has a linear mass density of 9 g/cm. String B has linear mass density of 18 g/cm. If the tension in both strings is the same, how far must string B be for it to be at resonance with the string A?. (07 Marks)

10. (a) State the law which used to describe the direction of force produce by a current carrying conductor. (04 Marks)

(b) Why the induced electromotive force (emf) is called the back electromotive force (emf)? (04 Marks)

(c) Two transformers A and B are step-up and step-down transformers respectively. Transformer A is at power station and transformer B is at consumption area Calculate the value of V4 given that;

N1 = 10, N2 = 200, N3 = 100, N4 = 50, and V1 = 240V. (07 Marks)

11. ( a) Explain why is the collector of the transistor made wider than emitter and base.(04 Marks)

(b) With the aid of diagram, explain the difference between conductors, Semiconductors and insulators based to their energy band. (06 Marks)

(c) The diagram below shows a puzzle box contains two lamps and simple components so that when T1 is connected to the anode lamp L1 light but when terminal T2 connected to the anode lamp L2 light. Suggest what the puzzle box is and how the connections are made.

image

FORM FOUR PHYSICS EXAM SERIES 171  

FORM FOUR PHYSICS EXAM SERIES 171  

THE UNITED REPUBLIC OF TANZANIA PRESIDENT'S OFFICE REGIONAL ADMINISTRATION AND LOCAL GOVERNMENT

FORM FOUR REGIONAL MOCK EXAMINATION 

TABORA REGION

031/1 PHYSICS -1

(For Both School and Private Candidates)

Time: 3 Hours Friday, May 12, 2023, a.m.

INSTRUCTIONS

  1. This paper consists of three sections A, B and C with a total number of eleven (11) questions.
  2. Answer all questions in sections A and B and two questions in section C.
  3. Section A carries sixteen (16) marks, section B carries fifty-four (54) marks and section C carries thirty (30) marks.
  4. Mathematical table and non-programmable calculators are allowed.
  5. Cellular iphone and any unauthorized materials are not allowed in the examination room.
  6. Write your examination number on every page of the answer sheet provided.
  7. You may use the following constants:
  • g = 10N/kg
  • Density of water = 1g/cm3 or 1000kg/m3
  • Pie (π) =3.14

SECTION A: (16 MARKS)

Answer all questions in this section

1. For each of the following items (i) to (x), choose a correct answer from the given alternatives and write its letter beside the items number in the answer booklet provided.

i. A form four student was given sets of energy so as to choose which among these energy cannot be re-used

  1. Fossils, sun, oil and nuclear
  2. Water, wind, wood and natural gas
  3. Oil, natural gas, and wood
  4. Natural gas, water, nuclear and wood
  5. Geothermal energy

ii. Form four students were discussing on the properties of matter, where one of them said that solid has definite shape and all members of the group agreed. Which one could be the reason behind for solid to have definite shape?

  1. It has high adhesive force 
  2. It has high surface tension
  3. It has low viscosity 
  4. It has high cohesive force
  5. It has low adhesive force

iii. ANGINJA was in the car, she tried to look at her friends who were outside of the car through a glass window, but she did not see them well. You as a form four student, what conclusion could you make on that glass window?

  1. It is transparent material 
  2. It is translucent
  3. It is opaque 
  4. It is not cleaned
  5. It is black

iv. Suppose you are given a positively charged rod, and you have placed that rod nearby side P of a neutral body PS. How will charges be distributed in the neutral body PS?

A
B
C
D
E

v. Suppose a singer is singing and the audience are enjoying the moment when a singer raises the voice and when the sound wave is lowered, Basing on the features of the sound, what is the feature of the sound that describes the sound given?

  1. Intensity 
  2. Beat
  3. Timber 
  4. Pitch
  5. Reverberation

vi. A red tie was viewed by using light from a torch which was blue. The colour that was seen was black. What happened to the blue colour from a torch when it met with red tie?

  1. Reflection
  2. Absorption
  3. Transmission
  4. Refraction
  5. Diffraction

vii. Ntanguye was cooking ugali in a good conducting container, but she seems to use iron handle which is covered by plastic at its holding handle to hold a cooking container. Why did she use plastic handle and not iron?

  1. It is good conductor of heat
  2. Its particles are closely to each other
  3. It reflects heat
  4. It is a poor conductor of heat 
  5. It is a good heat emitter

viii. One of the bulbs connected in the house seems to give out a bright light than other bulbs. Which device could be used to know the amount of potential drop used by a bulb

  1. Rheostat
  2. Ammeter
  3. Galvanometer
  4. Multimeter
  5. Voltmeter

ix. An atom was unstable, so it underwent disintegration, as a result different rays were released but one among those rays had negative charge. Which rays do you think might be the one?

  1. Gamma rays
  2. X-rays 
  3. Cathode rays 
  4. Beta particles
  5. Alpha particles

x. An astronomer observed group of stars that formed definite shape or pattern when viewed from the Earth, what name is given to those stars?

  1. Constellations 
  2. Comets
  3. Asteroids
  4. Meteoroids
  5. Satellite

2. Match the following concepts in List A with the relevant description in List B by writing its letter beside the item number in sheet provided.

List A List B

(i) Self-induction 

(ii) Lenz's law 

(iii) Mutual induction 

(iv) Step down transformer 

(v) Faraday's law of electromagnetic induction 

(vi) Maxwell's cork screw rule 

  1. Determines direction of the magnetic field produced by current carrying conductor 
  2. Relates the magnitude of induced e.m.f. and rate of change of magnetic flux linking a conductor 
  3. Describes the direction of the induced e.m.f. around the closed loop 
  4. Variation of the magnitude of current flowing in a conductor itself 
  5. Variation of the magnitude of current in one coil induce current in the other 
  6. Secondary voltage is smaller than primary voltage 
  7. Primary voltage is higher than secondary voltage 
  8. Interaction between electric and magnetic field 

SECTION B: (54 MARKS)

Answer all questions in this section

3. (a) Plane mirror forms an image of different features. By using a diagram show how a plane mirror forms an image of its features. (04 marks)

(b) Dentists prefer to use a concave mirror, in their work. With a help of a diagram explain why. (05 marks)

4. (a) NYAMWERU was at home cultivating. He had two hoes, sharp and blunt hoe. Blunt hoe was not cutting well as how sharp hoe did. Explain to him why sharp hoe cuts well than blunt hoe. (03 marks)

(b) A cube of sides 2cm is completely submerged in water so that the bottom of the cube is at a depth of 10cm. find:

  1. Difference in pressure between bottom and top of the cube. (3.5 marks)
  2. Different of force between bottom and top of the cube. (2.5 marks)

5. (a) AFYUSISE lived in a street where there are no cars. . When he visited his brother who lives in Town he saw cars moving around and he started to ask himself some questions:

  1. Why the car has circular wheel?
  2. Why the car tires are grooved? and
  3. Why the car engine is direct over the driving wheel?

Now you as a form four student help him to understand the reasons behind each question he asked himself: (06 marks)

(b) An athlete exerts a force of 100N while running 100m race, if she uses 50,000J of food energy. Calculate her efficiency, (03 marks)

6. (a) Matter expands when heated and contracts when cooled, by using a diagram of a ball and ring apparatus, explain how solid expand when heated. (05 marks)

(b) When hot water was poured to put off the fire, some hot water dropped to the people and caused less burning compared to the burning that was caused by vapour. Explain why vapour caused severe burning. (05 marks)

7. (a) Russia-Ukraine war is in tension of using weapons of mass destruction. Explain three health effects that are likely to occur if weapons of mass destruction will be used. (03 marks)

(b) Among the properties of cathode rays is that "when stopped suddenly they form an X-rays". With the help of well labeled diagram explain how cathode rays produce X-rays. (06 marks)

  1. (a) An increase in temperature to the environment has become a global problem. Analyze five solution to this problem (05 marks)

(b) A physics teacher guided the students to draw an electrical circuit which has a total resistance of 3Ω. Unfortunately in the school laboratory there were three (3) resistance of 2Ω each. Explain how will a student form the desired resistance and draw its circuit. (04 marks)

SECTION C: (30 MARKS)

Answer only two questions from this section

9.(a) Explain why does a solid weigh more in air than when immersed in a liquid? (04 marks)

  1. By using a help of diagram explain what happen to the two parallel straight conductors when current is moving in the same direction and in opposite direction. (04 marks)
  2. A block of wood of mass 5kg is placed on a rough inclined plane, at 60° to the horizontal. If the coefficient of kinetic friction between the wood and the plane is 0.3, determine the acceleration of the wood down the plane. (05 marks)

10. (a) (i) When in forward bias a PN junction conducts electric current but when in reverse bias does not conduct electric current, Describe this statement. (08 marks)

(ii) A teacher asked learners to mention example of insulator, learners mentioned wood, Give two reasons why wood is an insulator. (03 marks)

(b) (i) A thorium nucleus has a symbol of 23490Th , it emitted two beta particles in succession, to form Radium (Ra). Write down the decay equation. (02marks)

(ii) Gamma rays and X-rays have the same common properties. What are two reasons that make them not the same rays? (02 marks)

11. (a) (i) Two turning forks of the same frequency are available in the laboratory. One of the forks is loaded with some plasticine to alter its frequency slightly. What would you expect to observe if the two forks are sounded together? (04 marks)

(ii) Musicians place their fingers at different lengths of a stringed instrument when using it. Explain why? (03 marks)

(b) (i) A string of length 40cm is made to vibrate at its fundamental frequency and then at second overtone. Calculate the wavelength in each case. (04 marks)

(ii) Explain why a singer who is singing in a hall of many people is heard more clearly than when sings in a hall without any person. (04 marks)

FORM FOUR PHYSICS EXAM SERIES 149  

FORM FOUR PHYSICS EXAM SERIES 149  

PRESIDENT'S OFFICE

 REGIONAL ADMINISTRATION AND LOCAL GOVERNMENT

LINDI REGION

FORM FOUR MOCK EXAMINATIONS

PHYSICS 1

033/1 

 3.00 Hours                                                                                              May 2023

INSTRUCTIONS

  1.                This paper consists of sections A, B and C with total of (11) questions
  2.                Answer all questions in sections A and B and any two questions in section C
  3.                Write your examination number on every page of your answer sheets.
  4.                All diagrams must be in pencil and all writing in ink.
  5.                Show your work clearly including neat diagrams
  6.                Where necessary use the following constants:

(i) Acceleration due to gravity,

(ii) Density of water, 

(iii) Specific heat capacity of water, 

(iv) Refractive index of water,

(v) Speed of sound in air, 

(vi) Speed of light in air, 

(vii) Speed of sound in water, 

(viii) Latent heat of fusion of ice,

 

SECTION A (16 MARKS)

 

  1.                For each of the items (i) - (x), choose the correct answer from among the given alternatives and write its letter beside the item number

(i) The property which distinguishes longitudinal waves from the transverse waves is the

  1.               Ability to be refracted
  2.               Need for a medium to travel
  3.               Relative direction of oscillations and propagations
  4.              Wavelength
  5.                The speed of propagation

(ii) Which of the following do not affect the rate of evaporation of water in a dam.

  1.               Surface area
  2.               Depth
  3.               Humidity
  4.              Barometric pressure
  5.                Temperature

(iii) A conductor becomes a magnet when

  1.               It is wrapped with a coil of wire
  2.               A soft iron core is used
  3.               An electric current flows through it
  4.              It is resistance is increased
  5.                It is passed through an electric field

(iv) Racing cars rarely get accidents despite of their high speed because

  1.               Have greater momentum
  2.               Have big tyres with big treads
  3.               Have wide based and low centre of gravity
  4.              Exert greater frictional force
  5.                Have less mass

(v) When a bus starts or stops moving, passengers tend to be jerked forward or backward, this is due to

  1.               Newton's First law of motion
  2.               Newton's second law of motion
  3.               Newton's third law of motion
  4.              Newton's law of universal gravitation law of motion
  5.                Momentum of the bus

(vii)Retina in human eye has the same function as which part of the lens camera

  1.               Shutter
  2.               Diaphram
  3.               Film
  4.              Convex lens
  5.                Adjusting knob

(viii) In the circuit shown in Figure 1 the batteryand the ammeter have negligible

 internal resistance. What will be the ammeter reading?

  1.               0.4 A
  2.               0.5 A
  3.               1.6 A
  4.              2.0 A
  5.                2.5 A

(viii) A body moved upward a distance of 20m. Calculate the time taken to reach the maximum height

  1.               5 seconds
  2.               10 seconds
  3.               15 seconds
  4.              2 seconds
  5.                20 seconds

(ix) Which factors influence friction between the surface of the road and tyres of a car moving with a constant speed

  1.               Weight and speed
  2.               Nature of the surface and weight
  3.               Surface area of the tyres and the speed
  4.              Acceleration and nature of the surface
  5.                Speed and nature of the surface

(x) Which of the following is not a scalar quantity?

  1.               Electric current
  2.               Work done
  3.               Speed
  4.              Momentum
  5.                Power
  1.                Match the property of the mirror in list A with their corresponding mirrors in list B, by writing the letter of the correct response beside the corresponding item number

 

ListA

List B

(i) Has wide field of view

(ii) Used as shaving mirrors

(iii) Forms laterally inverted images

(iv) They have spherical shape

(v) Are used in periscope

(vi) Are used in car head light

  1.               Convex mirror
  2.               Plane mirror
  3.               Parabolic mirror
  4.              Concave mirror
  5.                Curved mirror
  6.                Inclined mirror
  7.              Rotating mirror
  8.              A coustic mirror

 

SECTION B (54 Marks)

  1.                (a) Briefly explain, when does a force become weight?

(b) A car of mass 1200 kg is brought to rest by a uniform force of 300N in 80 seconds. 

What was the speed of the car?

(c) A rectangular log of wood of density  has dimensions

(i) Calculate the maximum pressure it can exert on the ground. How is it experienced?

(ii) Calculate the minimum pressure it can exert on the ground. How can this be 

observed?

  1.                (a)  A mechanical  engineer prefer to use  long spanner in loosing a tight nut than a short one.

(i) State the principle that govern the process

(ii) Explain why engineer choose to do so in 4(a) above.

(b) A 2.0N weight placed on a10 cm mark of a metre rule just balances an object hanging from the 60cm mark when pivoted at the middle. Calculate the weight of the object

  1.                (a) Briefly give reasons for the following

(i) The fundamental frequency may alter during the day

(ii) Bat is able to fly at night without colliding with the wall

(iii) Two distinct sounds are heard by the person standing on a railway line when 

sounded at distance.

(b) The frequency obtained from a plucked string is 400Hz when the tension is 2N. 

Calculate

 (i) The frequency when the tension is increased to 8N.

 (ii) The tension needed to produce a note frequency of 600Hz.

 

  1.                (a) A compound bimetallic strip of brass and iron is straight at room temperature. Draw a labelled diagram to show how its appearance will be when it has been

(i) Heated to a high temperature

(ii) Cooled below 

(c) Explain why electrical cables and telephone lines are left bending downward when hanging.

 

  1.                (a) (i) Explain why sun is seen is moving from east to west.

 (ii) In building construction Architecture wants to add two forces in order to obtain the resultant of two force. Identify two laws which guide the 

architecture and state them.

(b)  (i) Explain why it is easy to keep moving body than to start stationary body to move.

 (ii) A body of mass 40kg is placed in a straight track inclined at an angle of  to horizontal. If the body is held from shipping by friction, calculate normal 

reaction and friction force.

 

  1.                (a) With the aid four (4) examples explain the term optical instruments.

(b) Distinguish between primary and secondary colours of light.

(c) Draw a diagram to show image formation by a compound microscope.

 

SECTION C (30 MARKS)

Answer only two (2) questions in this section

  1.                (a) Name apparatus used in the following energy transformation process.

(i) Mechanical energy to electrical energy

(ii) Electrical energy to mechanical energy

(iii) Electrical energy to sound energy

(iv) Electrical energy to light energy

 

(b) A 2kg object is rest on a table 1.2m above the floor. The ceiling in one room is 2.8m

 above the floor as shown in figure 2

 

 (i) Calculate gravitational potential energy of the object relative to the table top.

 (ii) Calculate gravitational potential energy of the object relative to the floor.

 (iii) Find the work done in lifting the object from the top of the table to the 

ceiling.

(c) Briefly explain how geothermal energy can be harvested

  1.           (a) State the following rules

(i) Cork screw rule

(ii) Dynamol rule

(iii) Fleming's left hand rule

(b) Carefully study figure 3which shows a design for an electrical operation model for lifting metal objects. Briefly explain three things you can do so that a heavier iron metal block can be lifted

(c) Explain how mirage seen by a motorist as a pool of water on the road is formed.

 

  1.           (a) Distinguish between the following terms as they used in waves

(i) Ultrasonic and infrasonic

(ii) Harmonic and overtone

(b) (i) How resonance occurs?

 (ii) In a closed pipe, the first resonance is at 23cm and second at 73cm. Determine the wavelength of the sound and the end correction of the pipe.

(c) Figure 4 shows the main bands of the electromagnetic spectrum. Region represents visible light.

 

 (i) Which region represents radiation capable of promoting the production of 

vitamin D in the skin?

 (ii) Which region contains radiation using in RADAR system?

 (iii) Which region contains radiation produced in nuclear reactor? 

 (iv) Find wavelength of radiation F, if its frequency is 150MHz.

 



FORM FOUR PHYSICS EXAM SERIES 144  

FORM FOUR PHYSICS EXAM SERIES 144  

IMG_20230509_084505NEMBO PRESIDENT’S OFFICE

      REGIONAL ADMINISTRATION AND LOCAL GOVERNMENT                       

MVOMERO DISTRICT COUNCIL

FORM FOUR MOCK EXAMINATION

 

 

031                                                                                                              PHYSICS 

Time: 3 Hours                                                                                                                       May 2023

 

Instructions

  1.                This paper consists of section A, B and C with a total of eleven (11) questions.
  2.                Answer all questions in section A and B and two (2) questions from section C.
  3.                Section A carries fifteen (15) marks, section B sixty (60) marks and section C carries twenty five(25) marks
  4.                Non programmable calculators and mathematical tables may be used.
  5.                Write your name or number on every page of your answer sheet.
  6.                Where necessary the following constants may be used.
  •               Acceleration due to gravity, g=10 m/s2 or 10 N/kg
  •               Density of water =1.0 g/cm2
  •               Pie,=3.14
  •               Speed of sound is 340 m/s

 


SECTION A (15 Marks)

Answer all questions in this section

  1.                For each of the items (i)-(x), choose the correct answer from among given alternatives, and write its letter beside the item number in the answer booklet provided. (10 marks)
  1.               A stone is thrown horizontally from the top of a building at a speed of 12 m/s. how long does it take to fall 45 meters vertically? (Assume negligible air resistance)
  1.               1.0 sec              B. 5.0 sec      C. 3.0 sec        D. 8.0 sec      E. 2.14 sec

 

  1.             In a race competition, one should kneel down when starting the race so that
  1.               No one can fall down by reaction produced by the earth to them
  2.               Each one can receive an acceleration which the earth provides
  3.               One of the rules in sports is not violated by anyone
  4.              Equal and opposite forces can act on a body to produce a resultant force.
  5.                Each one could exert a force on the surface of the ground
  1.            Heat would be lost in the thermos flask if the walls of the glass container were not coated with silver. Which process contributes to this kind of heat loss?
  1.               Radiation
  2.               Conduction
  3.               Convection
  4.              Absorption
  5.                Transmission
  1.            In a light experiment, observing an object through a certain material showed that less light was transmitted and the image was distorted. Which type of material was used
  1.               A translucent material
  2.               An opaque material
  3.               A luminous material
  4.              A transparent material
  5.                A non-luminous material

 

  1.             Why oil is used as lubricant
  1.               It has low density
  2.               It is highly viscous
  3.               It is flammable
  4.              It is less viscous
  5.                It is costless
  1.            When silicon element is doped with phosphorus atom, it produces:
  1.               A P-type semiconductor
  2.               Force
  3.               An intrinsic semiconductor
  4.              A bipolar semiconductor
  5.                An n-type semiconductor

 

  1.          A body is said to be in equilibrium if
  1.               It moves with uniform speed
  2.               The net force acting on it is zero
  3.               The upward and downward forces are equal
  4.              Its center of gravity is low positioned
  5.                Its center of gravity is high

 

  1.       What happens when a liquid changes into gaseous state?
  1.               Some surface molecules absorb latent heat of vaporization and escape
  2.               It gives its own latent heat that can be used to heat up the surrounding
  3.               The potential and kinetic energies of the molecules increase
  4.              The molecules attractive forces to one another increases and their average kinetic energy decreases
  5.                There is no adhesive force between molecules.
  1.           What is the main function of step up transformer?
  1.               To change a.c to d.c current
  2.               To decrease resistance in a circuit
  3.               To increase a.c voltage
  4.              To decrease a.c voltage
  5.                To increase a.c current
  1.              Which of the following arguments describes the mathematical language used in physics?
  1.               Density is mass per volume
  2.               Matter occupies space
  3.               Volume is the amount of space occupied by the body
  4.              Physics is the branch of science
  5.                Action and reaction are equal and opposite

 

  1.                Match the items in List A with response in List B by writing the letter of the correct response beside the item number in the answer booklet provided.(05 marks)

List A

List B

  1.               Longitudinal wave
  2.             Reverberation
  3.           Snell’s law
  4.            Critical angle
  5.             Principle of superposition
  1.               Sin r over sin i
  2.               Refractive index
  3.               Water waves
  4.              Sound wave
  5.                Angle of incidence for which the angle of refractive is 90°
  6.                Rise and fall of sounds of two vibrating objects
  7.              The resulting displacement at any point is equal to the sum of displacements of different waves
  8.              The multiple reflection of sound waves
  9.                  Angle of reflection for which the angle of incidence is 90°
  10.                 The resulting of the wave length


SECTION B (60 Marks)

Answer all questions in this section

  1.                (a) Form one students from Mzumbe secondary school visited Kilimanjaro Mountain. When climbing the Mountain to high altitude, one of the students got the problem of nose bleeding.
  1.               Comment on why student got such problem?( 02 marks)
  2.             Why astronauts wear space suits?(02 marks)

 (b)  The acceleration due to gravity on Jupiter is about 2.6 times that on the earth. A spacecraft has a weight of 24500 N on earth.

  (i) What is the mass of the spacecraft?(03 marks)

  (ii) What would be its weight on Jupiter?(03 marks)

  1.                (a) What is the index if refraction for a certain medium, if the light in air enters the medium at an angle of 30° and refracted at 22°?( 04 marks)

 

(b) A vertical object 10 cm high is placed 20 cm away from a convex mirror of radius of curvature 30 cm. using a ray diagram, determine (06 marks)

    (i) Image distance

    (ii) The height of the image formed

  1.           The magnification of the image

 

  1.                (a) (i) What do you understand the term Specific heat capacity and Specific latent heat of vaporization( 02 marks)

     (ii) Explain the factors which affect the boiling point of water.(03 marks)

(b) A sample contains 800 g of Iodine-131. How much Iodine-131 in the sample will remain undecayed after 40 days, If the Half-life of Iodine-131 is 8 days?(05 marks)

6.  (a) Mr. Samwel put the coin on a card placed over the mouth of a bottle. When the card is flicked away with the figure the coin drops neatly into the bottle.

 (i) Which law demonstrated by Mr.Samwel?(01 mark)

 (ii) State the law identified in (i) above(02 marks)

(b)   A screw jack has a screw pitch of 5mm and the effort arm 16 cm 

 (i) State two forms of energy in which the energy supplied to the screw jack is finally converted     to (2 marks).

(ii) Determine the percentage efficiency of this screw jack, if it needs an effort of 30N to lift a     load of 750N. (04 marks)

7.  (a)  Describe how a lens camera operates the same as human eye. Give three points (06 marks)

 (b)  Briefly explain how conduction of heat can be applied in your daily life(three reasons)(04 marks)

  1.                Weather forecasting shows that there is the possibility of occurring earthquake in Mtakuja village. The villagers are not aware with such disaster. As physicist, make villagers aware with
  1.             Meaning of earthquakes( 02 marks)
  2.            The four key indicators observed before an earthquake occurs (04 marks)
  3.              Suggest four precautions to be taken by villagers during an earthquake in order to minimize injuries and deaths. (04 marks)

SECTION C (25 MARKS)

Answer two (02) questions in this section.

  1.                (a) George Ohm observed that as the current flows through the circuit, it encounters some opposition. This opposition determines the amount of current flowing in electric device depending to the particular material.

(i) State the law that Mr. George formulate.(02 marks)

 (ii) Briefly explain factors affecting resistance of a conductor observed by Mr. George Ohm to sum up his observation. (04 marks)

 (b) (i) Distinguish between the concept of conductors, semiconductor and insulators in term of energy bands(04.5 marks)

 (ii) Give out one structural difference between A.C and D.C generators.(02 marks)

 

  1.           (a) Briefly explain how length of the string affects frequency of vibrating string.(02 marks)

(b) A string has length of 75 cm and a mass of 8.2 g. the tension in the string is 18 N. What are the frequencies of the 1st and 3rd harmonics? (06 marks)

(c) A transformer is used to step down 240 V mains supply to 12 V for laboratory use. If the primary coil has 600 turns, determine the number of turns in the secondary coil.(04.5 marks)

11. (a) Why some semiconductors called P type and other called N type? (3 marks)

(b)Describe, how you will connect the semiconductor diode as forward bias.  (4 marks).

 (b) The jaws of a vernier caliper touch the inner wall of calorimeter without pressure. The position of zero of vernier scale on the main scale reads 3.4 cm. the 6th of vernier scale division coincides with the main scale division. Vernier constant of calipers is 0.01 cm. Find actual internal diameter of calorimeter, when it is observed that the vernier scale has a zero error of -0.03 cm.(5.5 marks)

Page 1 of 8

 

FORM FOUR PHYSICS EXAM SERIES 126  

FORM FOUR PHYSICS EXAM SERIES 126  

PRESIDENT'S OFFICE REGIONAL ADMINISTRATION AND LOCAL GOVERNMENT

KILIMANJARO REGION MOCK FORM FOUR EXAMINATION

CODE:031 PHYSICS

TIME 3:00 HOURS   MAY 2023

INSTRUCTIONS

1. This paper consists of sections A, B and C.

2. Answer ALL questions in section A and B and only two questions from section C.

3. All writings should be in blue/black ink and drawing should be in pencil

4. Whenever necessary, use the following constants

  • Acceleration due to gravity, g = 10m/s2
  • Density of water, ρW = lg/cm3
  • Coefficient of linear expansion of steel a= 1.05 X 10-3K-1

SECTION A

1. For each of the following items (i) — (x),Choose the correct answer among the given alternative and write its letter besides the item number in the answer booklet provided.

i. Which pairs of instruments is used for measuring length?

  1. A meter rule and a measuring cylinder
  2. A micrometer screw gauge and beam balance
  3. A vernier caliper and a micrometer screw gauge.
  4. A pipette and vernier caliper
  5. A beam balance and spring balance.

ii. A block of wood has a density of 0.6g/cm3 and volume of 25cm3.The mass of block in grams is ;

  1. 1.5
  2. 15
  3. 41.6
  4. 150
  5. 1500

iii. A blue dress with red sports when viewed in red light will appear

  1. Completely red
  2. Black with red sports
  3. Magenta with red sports 
  4. Blue with red sports
  5. Complete with magenta.

iv. A stone is dropped from the top of a building hit the ground 4 seconds later. What was its velocity when hitting the ground?

  1. 400m/s
  2. 45m/s
  3. 40m/s
  4. 4.5m/s
  5. 0.4m/s

v. If a body vibrates with a frequency of 20 vibrations per second to 20,000 vibrations per second, then its vibration are perceived by the human ear. These vibration are called

  1. Ultra sonic vibrations 
  2. Infrasonic vibrations 
  3. Audible range.
  4. High frequency radio waves
  5. Ultraviolet.

vie The law which relates temperature and volume of a gas at constant pressure is called;

  1. Boyle's law
  2. Pressure law 
  3. Gas law
  4. Charle's law
  5. The law of Buoyancy

vii. A rectangular box of mass 10kg rest on an inclined plane. If the coeffcients of static and dynamic friction are 0.55 and 0.25 respectively, At what angle will the box begin to slide?

  1. 14.50 
  2. 24.80
  3. 28.80 
  4. 38.80
  5. 48.80

viii. What is the name given to minor shocks received after a major earth quake

  1. Tsunami
  2. Aftershock
  3. Epicentre 
  4. Focus
  5. Seismograph.

ix. The principle of fluid pressure which is used in hydraulic breaks is the,

  1. Pressure is the same at all level in a fluid
  2. Increases of pressure are transmitted equal to all parts of the fluid
  3. The pressure at a point in a fluid is due to the weight of the fluid above it. 
  4. Increases of pressure can only be transmitted through fluid.
  5. The pressure at a given depth is proportional to the depth in the fluid.

x. A form one student was fetching water from the pond, he was surprised to see an insect walking on the water surface without sinking. This phenomenon was due to: 

  1. Capillarity. 
  2. Surface tension. 
  3. Osmosis. 
  4. Diffusion.
  5. Density.

2. Match the item in LIST A with their corresponding response in LIST B by writing the letter of the correct response beside the item number.

LIST A

LIST B

  1. Thermistor. 
  2. Light Emitting Diode(LED).
  3. Zener diode.
  4. Semiconductor diode. 
  5. Metal semiconductor diode.
  6. Transistor.
  1. Most used in rectification
  2. Used to amplify or switch electronic signals and electric power.
  3. Used in simple indicator lamps and huge display screen to optical fibre communication links
  4. Used as voltage regulator device
  5. Used to control maximum temperature of hot liquid.
  6. Used to store charge
  7. Used for very fast switching and microwave applications

SECTION B

3. (a) Briefly explain why are the doors and shelves of oven made loosely fitting?

(b) A steel bridge over a motor way is 20m long at 0 0C.How much longer is it at 20 0C?

4. (a) Briefly explain four factors which determine the resistance of a conductor.

(b) A cell of 6.0V e.m.f and negligible internal resistance is connected to a resistor and drives a current of 3.0A through it. Another cell of e.m.f 1.5V is inserted in the circuit in series with the first one. The current remains 3.0A what is the internal resistance.

5. (a) Practically efficiency of a machine is less than 100% comment on this statement.

(b) A block and tackle system of 5 pulley is used to raise a load 500N steadily though a height of 20m. The work done against friction is 2000J. Calculate

  1. The work done by the effort 
  2. The efficiency of the system. 
  3. The effort applied.

6. (a) Explain why a bat can fly in the dark without hitting objects?

(b) A soldier standing Infront of a vertical cliff fires a gun, He hears the echo after 3sec. On moving closer to the cliff by 82.5m, he fires again and hears the echo after 2.5sec. find

  1. The distance of the cliff from the initial position of the man.
  2. The speed of sound.

7. (a) Describe four applications of radio- isotopes

(b) (i) A sample has an activity of 800 count-rates/min and half life of 50days how long will the activity become 50 count- rates/ minutes? 

(ii) Complete and balance the equation.

8. (a) Explain three sources of energy loss of a transformer

(b) A transformer with primary and secondary winding of 200 turns and 100 turns respectively is connected to 250 V mains. Calculate the secondary voltage if the transformer is 75% efficient.

SECTION C

Answer only two questions from this section.

9. (a) (i) Describe four effects of volcanoes on the earth.

(ii) Briefly explain four precaution that should be undertaken against earth quake hazards.

(b) (i)Explain three factors that affect frequency of a vibrating string.

(ii) A string of length 1m and mass 5 x 10-4Kg fixed at both ends is under tension of 20N. It is plunked at a point situated 25cm from one end. What would be the frequency of vibration of the string.

10. (a) Describe the mechanism of doping an intrinsic semiconductor to obtain p-type Semiconductor.

(b) Why most of the transistor in use are n-p-n transistor?

(c) Form one student were shouting in their classroom.Briefly explain how other students in the next room can hear them shouting.

11. (a) With the aid of diagram explain the function of fuse in an electrical appliance. 

(b)A circuit in a house is protected by a 10A fuse. The circuit is connected to the 240V mains.The following appliance are connected to the circuit.

APPLIANCE POWER RATING 
Bulb 1 100 W
Bulb 2 75 W
TV 300 W
Heater 1500 W

Determine whether the fuse will blow. on or off if the appliance are turned on.

(c) An aluminum block of mass 2.1 kg rest on steel platform. A horizontal force of 15N is applied to the block.

i. Given that coefficient of limiting friction 0.6, will the block move?

ii. If will moves, what will be its acceleration if kinetic friction is 0.47

FORM FOUR PHYSICS EXAM SERIES 121  

FORM FOUR PHYSICS EXAM SERIES 121  

PRESIDENTS OFFICE REGIONAL ADMINISTRATION AND LOCAL GOVERNMENT

COAST REGION 

FORM FOUR SECONDARY EDUCATION MOCK EXAMINATION

Code: 031/1 PHYSICS

(For Both School and Private candidates) 

Time: 3 Hours Year 2023

INSTRUCTIONS

  1. This paper consists of sections A, B and C with a total of eleven (11) questions
  2. Answer all questions in section A and B and two (2) Questions from section C 
  3. Cellular phones and any Unauthorized materials are not allowed in the examination room
  4. Non — Programmable calculators may be used 
  5. Write your examination number on every page of your answer booklet (s) 
  6. Where necessary the following constants may be used 
  •  Acceleration due to gravity, g=10m/s2 
  • Density of water = 1000 kg/m3 
  • Speed of light in air= 3.0 x108 m/s 
  • Pie (π) = 3.14 
  • Specific heat capacity of water = 4200J/Kgk

SECTION A (16 Marks) 
Answer all questions in this section

1. For each of the items (i— x) choose the most correct answer among the given alternatives and write its letter beside the item number in the answer booklet (s) provided.

(i) A bus carrying a very big load on its carrier can easily overtone because

  1. Its center of gravity is low 
  2. Its center of gravity is high
  3. Its equilibrium is stable 
  4. It cannot run fast
  5. It is more stable and can run fast

(ii) Any phenomenon which repeats itself to a regular internal of time can be used as a measure of time. Which among of the following phenomena is not likely to be?

  1. Oscillation of pendulum
  2. Revolution of the moon around the earth
  3. Rotation of the wheels of a moving sports
  4. Revolution of the earth around the sun
  5. Rotation of the earth about its axis

(iii) A spiral spring of natural length 1.5m is extended to 1.505, by a force of 0.80N, what will be its extension when the applied force is 3.20 N?

  1. 0.020m 
  2. 0.005m 
  3. 0.20m
  4. 0.5m 
  5. 0.05m

(iv) The Sonometer wire is vibrating in the second over tone. We may say there are. 

  1. Three nodes and four antinodes 
  2. Too nodes and two antinodes
  3. One node and two antinodes 
  4. Two nodes and one antinode
  5. Four nodes and three antinodes

(v) Illuminated smoke particle suspended in air are viewed with a microscope and are seen to vibrate continuously. What do you think this vibration o the smoke particle is caused by?

  1. Shaken by the vibration of the molecules within them
  2. Agitated by sound waves from the surrounding room
  3. They are bombarded continuously by air molecules
  4. Move around by convection currents
  5. Move around by convection current.

(vi) High voltage is used for transmitting electricity on the National GRID, this because high voltage.

  1. Needs transformer for conversion
  2. Would facilitate power distribution of customers
  3. Means high current would be used
  4. Is needs everywhere
  5. Would minimize electrical energy losses by using low current

(vii) A body of 10N in air and SN when completely immersed in water, neglecting the up thrust in air, its Weight in a liquid of density 1.5g/cm3 will be

  1. 2N 
  2. 3N 
  3. 7N
  4. 18N 
  5. 10N

(viii) Which of the following letter is correct referring to ampere's rule of direction of magnetic field?

Figure 1

  1. P
  2. Q
  3. R
  4. P and R
  5. All

(ix) A white shawl wrapped around a baby keeps the baby warm because the shawl;

  1. Conduct heat to the baby 
  2. Is an insulator
  3. Is a poor reflector 
  4. Stop convection currents
  5. Is a poor radiator

(x) The pressure of a liquid contained in a jar depends on

  1. Density of the liquid 
  2. Depth of the liquid in jar
  3. Acceleration due to gravity 
  4. Both density and depth of the liquid
  5. Volume of the liquid and acceleration due to liquid

2. Match the items in List A with responses in List B by writing the letter of the correct response beside the item number in the answer booklet provided.

List A

List B

(i) It can be used to find out about the depth of the sea

(ii) Occurs when the applied frequency of the pushes is the same as natural frequency of the object.

(iii) Depends on the amplitude of the waves.

(iv) Varies directly proportional to the frequency of the sound.

(v) Provide vitamin D in human skin.

(vi) Have shorter wavelength

  1. X-rays
  2. Echo
  3. Ultra violet
  4. Pitch
  5. Resonance frequency
  6. Wavelength
  7. Sound wave
  8. Optical fibres
  9. Loudness

SECTION B (54Marks) 
Answer all questions in this section.

3. (a) Briefly explain why letters on the front of an ambulance are written laterally inverted as a  (4 marks)

(b) A compound microscope consists of two lenses of focal length 12 cm and 6 cm for the objective lens and the eyepiece lens respectively. The two lenses are separated by a distance of 30 cm. The microscope is then focused to a point where the image is forme_d at infinity. Use lens formula to determine the position of the object. (5 marks)

4. (a) A pulley system machine is used to lift a body of weight 400N when an effort of 100N is applied. If the efficiency of this machine is 80%, draw a sketched diagram of this machine (5 marks)

(b) Uniform metal beam of length 5 m and mass 9 kg is suspended horizontally by two wires attached at 50cm from the left end of the beam and 150 cm from the right end of the beam. Furthermore, loads of 60N and 150 N are placed at the quarter and three-quarter length of the beam respectively, from the left end. Determine the tension in each wire? (4 marks)

5. (a) You are provided with two materials of the same looking, one is conductor and the other in semiconductor. Give two experiments in which you can distinguish between them. (4 marks)

(b) A string is set vibrating by plucking at a point one-quarter of the length from the end. Derive the expression of the frequency note produced by the vibrating string. (5 marks)

6. (a) An ice forms at the top of ocean and other water bodies during freezing condition but not at the bottom. Identify the name given to this phenomenon and the significance of this phenomenon. Hence draw the graph of density against temperature that leading to the phenomenon. (05 marks)

(b) The specific heat capacity of water is 42003/kg/c what does this statement mean?, If a student of mass 50kg wanted to take a bath mixed 4kg of water at 80°C with 6kg of water at 20°C. What is the final volume and final temperature of the water (4 marks)

7. (a) Briefly explain five (5) effects of global warming in our environment. (04 marks)

b) (i) Although Saturn is lager planet than Earth but it has less gravity than the earth. Explain why? (2.5 marks)

(ii) Explain the force that keeps the earth in its orbit. Hence state the law which gives the magnitude of this force. (2.5 marks)

8. (a) X- rays are just like features to be defined, X-ray tube is just like women and her amniotic sac and cathode is just like a men and its ray is just like sperms. With the aid of diagram describe the statement. Hence give reason why the tube should be evacuated. (05 marks)

(b) Danger sign along the road as well as tail and brake lamps of motor vehicles rear are painted red and not blue. Briefly explain the reason behind. (04 marks)

SECTION C (30 Marks) 
Answer only two questions from this section.

9. (a) With the aid of diagram explain how full wave rectifiers operate? Hence give reason why is most preferred in electronic circuit? (05 marks)

(b) Briefly explain as to why transformer cannot work with d.c voltage? (O5marks)

(c) A power generator produces an e.m.f of 33,000V at a frequency of 50HZ. The domestic supply is approximately 250V, 50HZ. Explain how the output of the power station can be modified for use in our homes? (05 marks)

10. (a) Briefly explain with two (2) strong reasons why NPN transistor is most preferred rather than PNP transistor? ( 05 marks)

(b) With the aid of diagram briefly explain why the forward biasing of the PN- junction allow electric current to flow possible? (05 marks)

(c) Give reason for the following cases.

  1. Fuses are made up of very thin wires but heaters are made up of thick wires (02 marks)
  2. Petrol tankers are painted white and not black. (02 marks)

11 (a) We say that acceleration due to gravity is constant only if we ignore air resistance. For which situations can air resistance be ignored? (Give 3 situations) (05marks)

(b) Explain why water is not used as a barometric liquid? (05 marks)

(c) If a high power heater is connected to mains, then the bulbs in the house become dim. Why? (05 marks)

FORM FOUR PHYSICS EXAM SERIES 113  

FORM FOUR PHYSICS EXAM SERIES 113  

TANZANIA HEADS OF ISLAMIC SCHOOLS COUNCIL

 FORM FOUR INTER ISLAMIC MOCK EXAMINATION

031/1 PHYSICS 1

(For Both School and Private Candidates)

Time: 3 Hours Tuesday, 13th September 2022 a.m.

Instructions

1. This paper consists of sections A, B and C with a total of eleven (11) questions.

2. Answer all questions in sections A and B and any two (2) questions from section C.

3. Section A carries fifteen (15) marks, section B carries sixty (60) marks and section C carries twenty-five (25) marks.

4. Cellular phones and any unauthorized materials are not allowed in the examination room.

5. Non-programmable calculators may be used.

6. Write Your Examination Number on every page of your answer booklet(s).

7. Where necessary the following constants may be used.

  1. Acceleration due to gravity, g = 10m/s2.
  2. Pie, π= 3.14
  3. Density of water ∫=1000kg/m3 
  4. Speed of light waves = 3.0 x 108m/s.
  5. Velocity of sound in air, V = 340m/s.
  6. Specific heat capacity of water = 4200J/kgoC.
  7. Specific heat capacity of copper calorimeter = 390J/kgoC.
  8. Density of gold = 19.3g/cm3 and density of silver = 10.5g/cm3.
  9. Relative density of alcohol = 0.8
  10. Resistivity of copper wire = 1.68 x 10-8Ωm.

SECTION A (15 Marks)

Answer all questions in this section

1. For each of the items (i) to (x) choose and write the letter of the most correct answer in the answer booklet (s) provided.

(i) A teacher of Physics was doing a practical and she found that the reading of a burette to be 13.90cm3, 60 drawing pins each of average volume of 0.1cm3 were added to the water, the new reading now is:

  1. 13.96 cm3 
  2. 19.70 cm3 
  3. 19.69 cm3
  4. 19.90 cm3 
  5. 73.90 cm3.

(ii) Ten best students from Murubona International School were taken to China and they visited one of the lakes there and they found that pond skaters were walking on the surface of that lake. This is because of the:

  1. Surface tension of that water
  2. Upthrust of the water
  3. Density of that water
  4. Capillarity of that water
  5. Force of that water.

(iii) A crate of soda with mass 50kg will just begin to slide with constant speed down a rough ramp (slope) at 30o to the horizontal. The coefficient of friction which is static was:

  1.  0.6774
  2. 0.5774
  3. 200.0000
  4. 346.4000
  5. 0.5000.

(iv) It has been proved by scientists that as one goes far away from the earth’s surface, the density of air will:

  1. Decrease
  2. Increase
  3. Remain constant
  4. Become greater than its weight
  5. Be zero.

(v) Which amongst the following could be the best reason as to why the sky is believed to appear blue while being observed from the earth’s surface?

  1. Selective scattering of sunlight
  2. Regular reflection of sunlight
  3. Irregular reflection of sunlight
  4. Diffuse refraction of sunlight 
  5. Selective scattering of moon light.

(vi) A measuring cylinder is filled with oil. A girl says that the pressure at the bottom of the cylinder depends on: 

1. The height of the oil

2. The area at the bottom

3. The oil lubrication property

4. The oil density

Which of A, B, C, D, and E is correct?

  1. 1 and 3 
  2. 2 and 4
  3. 1 and 2
  4. 1 and 4
  5. 3 and 4.

(vii) A rectangular wooden block of density 0.8g/cm3 has dimensions 0.5m x 0.8m x 6m what maximum pressure will it exert on the ground?

  1. 48,000N/m2
  2. 4000N/m2
  3. 2.4N/m2
  4. 19,200N/m2
  5. 400N/m2.

(viii) One of Art’s students was found stretching a certain wire and that wire stayed inelastic. Explain as a physics student whether the force applied is: 

  1. Zero
  2. Independent of extension
  3. Directly proportional to extension
  4. Inversely proportional to extension 
  5. Dependent and independent of extension.

(ix) Normally it is seen practically that an airship is always floating high above the ground, it means in this case:

  1. The air upthrust is equal to airship weight
  2. The air temperature inside the airship is equal to the air temperature outside
  3. The air density outside the airship is equal to the air density inside
  4. The air temperature inside the airship is less than air temperature inside the airship
  5. The air temperature is always not varying.

(x) A nail is being pulled by Amina, a form three student at Yombo Secondary using a string from a wall. The string forms an angle of 30o with the nail. If the force being used is 10N, what magnitude of the force tends to bend the nail?

  1. 8.66N
  2. 0.866N
  3. 5N
  4. 0.500N
  5. 50N.

2. Match the descriptions of the uses of electromagnetic radiations in LIST A with the correct names of the waves in LIST B by writing a letter in the answer sheet provided.

LIST A

LIST B

(i) Electromagnetic radiation which are used to increase the rate of growth of chickens.

(ii) Electromagnetic radiation which are used by the global positioning satellite (GPS) to precisely pin point the location of the targets.

(iii) Electromagnetic radiations which are used for satellite communications.

(iv) Electromagnetic radiations which are used in CD and DVD players to read the content of the disk

(v) Electromagnetic radiations which are used in remote sensing of vegetation and for identification of different objects.

  1. Radio waves
  2. Micro waves
  3. Infra-red
  4. Visible light
  5. Ultra-violet
  6. X-rays
  7. Gamma rays

SECTION B (60 Marks)

Answer all questions in this section

3. (a) The Russia – Ukraine war has a tension of use of weapons of mass destruction. Mention three possible dangers associated with these weapons.  [06 marks]

(b) A mirage is often seen by a motorist as a pool of water on the road some distance ahead. With the aid of diagram, give an explanation of its formation.  [04 marks]

4. (a) Measurement is an essential tool in day to day life, briefly explain how measurement can assist the human being in the following areas:

(i) In architecture

(ii) In hospital [05 marks]

(b) A crown made of gold and silver has a volume of 60cm3 and mass of 1.05kg. Estimate the exact mass of gold contained in the crown. [05 marks]

5. (a) Amongst advantages of the diode is that, it is a suitable device for converting sinusoidal wave form into unidirectional wave form of electricity. Explain how is this done in real

life situation. [05 marks]

(b) Transistors are sincerely base elements of modern electronics and all the people have great interest with them. Suggest utmost five applications that transistors have to people.

 [05 marks]

6. (a) How is the principal focus of a convex mirror explained as far as light is concerned,

A concave mirror is used to form an image of an object pin where the object must be placed to obtain:

(i) An upright, enlarged image

(ii) An image the same size as the object. [05 marks]

(b) A diagram below shows the path of a ray of light through one corner of a cube ice. Find:

(i) The angle of incidence as the AB

(ii) The angle of refraction at this face. [05 marks]

7. (a) It is believed by physicists here in Tanzania and the whole world that we cannot verify ohm’s law by using a filament lamp. Explain why is this true? [3 marks]

(b) As a form four physics student do you think that electromotive force (e.m.f) is different from potential difference (V). Give three points (if any). [3 marks]

(c) What is the resistance of copper wire of length 20cm and diameter 0.080cm? [4 marks]

8. (a) Explain with one reason as why Tanzania ports in Dar es Salaam and Zanzibar block and tackle pulleys are commonly used. [2 marks]

(b) Exactly explain to why the efficiency of a pulley system is always less than 100%. Give only two reasons. [2 marks]

(c) A wheel and axle with an efficiency of 90% is to be used to raise load of 10,000N. The radius of the wheel is 40cm while that of axle is 5cm, calculate:

(i) The velocity ratio of the wheel and axle [2 marks]

(ii) The mechanical advantage of the wheel and axle [2 marks] (iii) The effort required to raise the 10,000N load [2 marks] 

SECTION C (25 Marks)

Answer only two (2) questions in this section

9. (a) Vectors are totally different from scalars in all aspects including direction. What do you think are two conditions to be satisfied for two vectors to be equal? [4 marks]

(b) State and explain practically the meaning of triangle law of vector addition. [2 ½ marks]

(c) Two vectors, one of 8N and the other 6N, are acting on a body. Given that the two forces are acting perpendicularly to each other. Find the magnitude of the third force which would just counter balance the two forces. [6 marks]

10. (a) Students of form four in Uganda were not taught about electronics. Suppose you were invited to speak to them, using a labelled diagram explain how full-wave rectification can be achieved by using two diodes and centre-tapped transformer. [6 marks]

(b) (i) Electrical energy is distributed in all parts of Tanzania by the National Grid

System which transmits a.c at very high voltage. Why is an a.c and not a d.c used? [2 marks]

(ii) Six cells each having an emf of 2V and internal resistance of negligible resistance, a 1.4Ω resistor and a metal-filament lamp. The ammeter reads 3A, what readings would you expect from a high resistance voltammeter connected across the battery terminals? [4 ½ marks]

11. (a) Discuss the meaning of the term “Laws of Stars”. [2 ½ marks]

(b) “Astronomy has a vital Role over thousands of years”. Elaborate four (4) ways to support that statement. [4 marks]

(c) What do you think are the three basic characteristics of planet until to date? [3 marks]

(d) What do you understand by the term constellation as far as astronomy is concerned and hence explain three of its uses in day to day life situations. [3 marks]

FORM FOUR PHYSICS EXAM SERIES 87  

FORM FOUR PHYSICS EXAM SERIES 87  

PRESIDENT'S OFFICE REGIONAL ADMINISTRATION AND LOCAL GOVERNMENT 
KILIMANJARO REGIONAL COMMISSIONER'S OFFICE 
FORM FOUR MOCK EXAMINATION

PHYSICS 1

CODE:31/1

TIME: 3 HOURS 01/07/2022 

INSTRUCTIONS 

  1. This paper consists of section A, B and C with total of eleven (11) questions.
  2. Answer all questions in section A and B and two (2) questions from section C.
  3. Section A carries fifteen (15) marks, section B sixty (60) marks and section c carries twenty five (25) marks.
  4. Cellular phones and any unauthorized materials are not allowed in the examination room.
  5. Non-programmable calculators and mathematical tables may be used.
  6. Write your Examination Number on every page of your answer booklet(s).
  7. Where necessary the following constants may be used:
  1. Acceleration due to gravity, g = 10 m s-2.
  1. Density of water, p = 1000 kg m-3 or 1 gcM-3.
  2. Pie, 7r. = 3.14.
  3. Specific latent heat of vaporization of water = 2.3 x 106 J / Kg

SECTION A (15 Marks)
Answer all questions in this section,

1. For each of the item (i) - (x), choose the most correct answer from 'among the given alternatives and write its letter beside the item number hi the answer booklet provided.

i. Which of the following matches is not correct in terms-of its SI UNIT?

  1. Mass — Kilogram
  2. Force Newton
  3. Length — Meter
  4. Time — Minutes
  5. Amount of substance - Mole

ii. A stone of mass 18g was immersed into a liquid and then removed. Figure 1 below shows initial liquid level, A, when the stone was fully immersed and the final level, B after the stone has been removed. Find the density of the stone.



Figure 1
  1. 2.25 g/cm3
  2. 1.80 g/cm3
  3. 2.00 g/cm3
  4. 9.00 g/cm3
  5. 12.00 g/cm3

iii. In a simple electric motor the commutator

  1. Connects the brushes together
  2. Reverses the magnet field
  3. Charges the current strength in the coil
  4. Changes the current direction in the coil.
  5. Reverse the battery poles.

iv. Kelvin scale has;

  1. One fixed point
  2. Two fixed points
  3. Three fixed points
  4. One fixed and two variable points
  5. None of the above

v. The resultant of two forces of 3N and 4N acting at right angle is equal to;

  1. 4N
  2. 3.5N
  3. 7,0N
  4. 3N
  5. 5N

vi. The cost of electricity for a 2000W electric fire used for 10;3.0 hours at the rate of Tshs 8 per KWh is;

  1. Tshs 168
  2. Tshs 80
  3. Tshs 64.8
  4. Tshs 1,680
  5. Tshs 8,000

vii. Oil is used as a lubricant in a machine because it has

  1. Low density
  2. High viscosity
  3. Low pressure
  4. High pressure
  5. Low viscosity

viii. A bar of copper is heated from 293K to 333K. Identify the false statement among the following

  1. Its density will increase slightly
  2. Its length will increase slightly
  3. Its electrical conduction will decrease slightly
  4. Its mass will not change
  5. Its weight will remain unchanged

ix. The graph below shows the decay curve of a radioactive substance. What is the half-life?



Figure 2
  1. 1 minute
  2. 2 minutes
  3. 3 minutes
  4. 4 minutes
  5. 5 minutes

x. The frequency of a wave increases when the speed remains the same. What happen to the distance between two consecutive crests?

  1. It increases
  2. It decreases
  3. Stay unchanged
  4. It increases and then decreases
  5. It decreases first and then increases

2. Match the items from List A with the corresponding labeled parts of the a.c generator in List B by writing the letter of the correct response beside the item number in the answer booklet provided.

LIST A

LIST B

(i) Are used to provide a constant magnetic field.

(ii) As it rotates, the magnetic field through it changes, which induces an e.m.f.

(iii) External circuit.

(iv) They turn with the coil. They are made of a good conductor such as copper.

(v) They keep electrical contact with the slip rings but do not move to the external circuit.

SECTION B (60 Marks)

Answer ALL questions in this section

3. (a) Figure 4 below shows rays of light AO, BO, CO incident on a glass --air interface. OA OP and OC are the corresponding emergent rays. Study and answer the questions that follow:



Figure 4

Determine the critical angle and refractive index of the glass material (06 marks)

(b) One of the common method used in our family to preserve Irish potatoes, is to put them in water. 
Explain briefly how this method preserve and keep potatoes health than when it is not used. (04 marks)

4. (a) A claw hammer can work as first class and third class lever. Justify this statement. (01 mark) 

(b) (i) Differentiate sound waves from electromagnetic waves. (04 marks)

(ii) Figure 5 below is a wave of string vibrating with a frequency of 500HZ. Determine 1) Amplitude 2) Wavelength 3) Speed of the wave



Figure 05

5. (a) Briefly explain the following phenomenon;

  1. A cool breeze blows from the sea on a hot summer day.
  2. Condensation of water vapour on a bath room mirror warms the bathroom

(b) A uniform half meter rule AB is freely pivoted at 10cm from end A and balance when a body of mass 35g is hanged at 2cm from end A.

  1. Sketch the diagram to illustrate the information above
  2. Calculate the mass of the ruler

6. (a) Using domain theory, explain the process of magnetization.

(b) The kettle contains 1.6kg of water, is left switched on, after starting to boil;

  1. How much heat energy will be used in turning all the water to steam.
  2. How long will it take for the 2.5kw kettle to boil dry. (06 marks)

7. (a) Give the most suitable type of electromagnetic radiation for the applications stated below.

  1. Satellite communications.
  2. Disinfecting water.
  3. Thermal imaging.
  4. Sterilizing food.
  5. Vision. (05 marks)

(b) A particular radioactive has a half— life of 2.0 hours. A sample gives a count rate of 2400 counts per second at 11:00 am. When will the count have dropped to approximately 300 counts per second in the same counting system? (05 marks)

8. (a) In the set up shown in figure 6 below a light copper ring is supported above the solenoid with a wooden peg. Explain why the copper ring becomes hot when the switch is closed.



Figure 06

(b) Briefly explain

  1. Meteorites
  2. Four (4) effects of global warming (04 marks)

SECTION C (25 Marks) 
Answer two (2) questions from this section.

9. (a) Give three reasons, why Tanzania decided to shift from analogue to digital signal transmission? 

(b) In only one of the given diagrams in figure 7 below, identify the circuits in which the lamp lights by giving a reason.(02 Marks)



Circuit A


Circuit B

Figure B

(c) The air in each of the four tyres of a motor car exerts a pressure above atmospheric pressure of 3 x 105 N/m2If each tyre is in contact with the ground over an area Of 80cm.2. Estimate the mass of the car. (05 marks)

10. (a) Wind energy is one of the sustainable energy sources with relative lOw running costs. Describe how electric energy is harnessed from wind energy. (05 marks)

  1. In a X- ray tube, explain why the filament is made of tungsten metal? (2.5 marks)
  2. (i) State the factors on which the strength of a transformer depends (02 marks)

(ii) A transformer steps down from 240V to 6V. If there are 1600tums on the primary, how many turns are needed on the secondary? If the transformer has an efficiency of 85% what current is taken from the 240V supply when 2A are taken from the secondary?

11. (a) The form three students at a certain secondary school perform an experiment to determine unknown resistance-of the wire X by using a meter bridge circuit. The apparatus arrangement used is as shown in the figure 8 below, where AB is a constant wire of length one meter. Find the resistance of wire X.



Figure 08

(b) The house is fitted and uses the following electrical appliances with their respective power rating as shown in the table below

AN ELECTRIC APPLIANCE

POWER RATING

4 bulbs

100 W each

Heater

1,200W .

Electric iron

1,000W

Television

400W

If each electrical appliance is used for 4 hours every day and the cost of using it is Shs 3501= per unit, find the cost of operating these items for 10 days. (5.5 marks)

FORM FOUR PHYSICS EXAM SERIES 38  

FORM FOUR PHYSICS EXAM SERIES 38  

PRESIDENT’S OFFICE REGIONAL ADMNINSTRATION AND LOCAL GOVERNMENT

LUDEWA DISTRICT COUNCIL

FORM FOUR MOCK EXAMINATION JUNE, 2022

031/1 PHYSICS 1

TIME: 3HRS Friday Morning, June 10th, 2022 a.m.

INSTRUCTIONS

1. This paper consists of sections A, B and C with a total of eleven (11) questions.

2. Answer all questions in sections A and B and only two (2) questions from section C.

3. Write your Examination number on every page of your answer sheet(s) or booklet.

4. Non-programmable calculators and four figures may be used.

5. Cellular phones are not allowed in the examination room.

6. Where necessary the following constants may be used;

(i) Acceleration due to gravity, g = 10m/s2 or 10N/kg

(ii) Density of water = 1g/cm3

(iii) Specific latent heat of condensation of steam (Lw) = 2.26 × 106 J/kg

(iv) Specific heat capacity of water (Cw) = 4200J/kgk

(v) Specific heat capacity of copper = 390J/kgk

(vi) Velocity of sound in air = 330 m/s

(vii) Standard atmospheric pressure = 101300 N/m2

SECTINS A (15 Marks)

Answer all questions

1.For each of the items (i) – (x), choose the correct answer from among the given alternatives and write its letter beside the item number in the answer sheet provided.

(i). The International system of units, SI has decided to use the wavelength of monochromatic light (wavelength of orange-red light emitted by the isotope krypton-86) as the standard of length.

What among the following is not true about the information above?

  1.  The wavelength of light is not affected by change in place
  2.  The wavelength of light is not affected by time
  3. The wavelength of light is not affected by temperature
  4. That kind of light is easily reproducible in any good laboratory.
  5. The wavelength of light is constant at standard temperature and pressure.

(ii) Any phenomenon which repeats itself at regular interval, can be used as a measure of time. Which among the following phenomena is not likely to be?

  1.  Oscillation of pendulum 
  2. Revolution of the earth around the sun 
  3.  Rotation of earth about its axis 
  4. Revolution of the moon around the earth
  5. Rotation of wheels of a very fast-moving sports car.

(iii). A car moving with a velocity of 40km/h can be stopped by applying brakes in 2m. If the same car is moving with the speed of 80km/h, what is the minimum stopping distance?

  1.  16 m 
  2. 12 m
  3. 8 m 
  4. 4 m
  5. 2 m

(iv) A clinical thermometer is used to measure the temperature of human bodies. During its construction, the biomedical equipment engineers prefer to use mercury as a thermometric liquid than water or alcohol. What do you think is the reason behind?

  1.  It is expensive 
  2. It is opaque and does not need coloring
  3.  It is more sensitive to temperature 
  4. It has high specific high heat capacity
  5. It is the most conducting and less dense than all other liquids.

(v) Figure 1 below shows a block being pulled along a track. If a force of 20N is applied in the direction A at an angle of 60º, what is the value of the force which tend to move the block in the direction of B?


Figure 1
  1.  17N 
  2. 12N
  3.  20N 
  4. 10
  5. 21N

(vi) A measuring cylinder is filled with oil. A girl says that the pressure at bottom of the cylinder depends on:

1. The height of the oil 

2. The area at the bottom 

3. The oil lubricating property

4. The oil density

Which of the following alternatives is not correct?

  1.  1 and 3 
  2. 2 and 4 
  3. 1 and 2 
  4. 1 and 4 
  5. 3 and 4

(vii) The monkey is descending from the branch of a tree with a constant acceleration. If the breaking strength of the branch is 75% of the weight of the monkey, what will be the minimum acceleration with which the monkey can slide down without breaking the branch?

  1.  g 
  2. 3g
  3.  g⁄2 
  4. g⁄4
  5. 4g

(viii) Illuminated smoke particles, suspended in air, are viewed with a microscope and are seen to vibrate continuously. What do you think this vibration of the smoke particles is caused by?

  1.  They are bombarded continuously by air molecules
  2.  Agitated by sound waves from the surrounding room
  3.  Shaken by the vibration of the molecules within them
  4.  They are supplied with energy by the light illuminating them.
  5. Moved around by convection currents

(ix) If no lubricating oil is used, the metal X of a piston rubs against metal Y of an engine casing. The amount of heat produced depends on:

  1.  Specific heat capacity of X 
  2. Frictional force between X and Y 
  3.  The specific heat capacity of Y 
  4. The conductivity of X and Y
  5. The mass of Y.

(x) The table below shows the saturation vapour pressure (S.V.P) of a liquid in centimetres of mercury.

Temp (℃)

20

30

40

50

60

70

80

S.V. P

30

44

52

68

74

78

84

At standard (normal) atmospheric pressure, the boiling point of the liquid is about:

  1.  35℃ 
  2. 40℃ 
  3. 55℃ 
  4. 65℃ 
  5. 80℃ 

2. Match the parts of a certain optical instrument from LIST A with the corresponding functions in LIST B by writing the letter of the correct response beside the item number in answer sheet provided.

 LIST A

 LIST B

Figure 2

  1. Magnifies and invert image so as to be seen upright on the screen.
  2. It inverts image side way
  3. Real image
  4. It diverges light to focus the image on the screen
  5. Virtual image
  6. It concentrates light which would otherwise be partly wasted.
  7. It is a doublet to reduce chromatic aberration
  8. Provides illumination
  9. It directs maximum amount of light from the image to the slide

SECTION B (60Marks)

Answer all questions

3. (a) You are with density bottle, water, oil and beam/ digital balance. Explain briefly the procedures you could use to determine the relative density of oil and deduce its formula. (6 marks) 

(b) A relative density bottle has a mass of 14.6g when dry and empty. Its mass is 58.1g when full of turpentine and 64.6g when full of water. Find the relative density (R.D) of turpentine. (4 marks)

4. (a) Repulsion is a sure test for electrification. As a form four student how can you explain this phenomenon? (5 marks)

(b) Although ordinary rubber is an insulator, the rubber tyres of an air craft are made slightly conducting. Explain why. (5 marks)

5. (a) “The specific heat latent heat of vaporisation of water is 2.26 × 106 J/kg”. The statement confused some students at a certain school. Assist them by giving the meaning of the statement. (3 marks) 

(b) 100g of dry steam at 100℃ is mixed with 2kg of water at 20℃. Determine the final temperature of the steam when all the steam has condensed. (7 marks)

6. (a) You are provided with two types of mirrors, concave mirror and convex mirror. What type of the mirror among the two will you prefer or driving a car to see the traffic at your back? Explain your choice. (4 marks)

(b) A far-sighted woman has a near point of 1.5m. Calculate the focal length of the lens for her ` eyeglasses so that she can read a book held at 25cm. Also find power of the lens.  (6 marks)

7. (a) A drum at station A is connected to a wire string at station B. A man at A beats the drum while another person at B places his ear at the wire and hears two sounds separated by the time interval of 0.5 seconds. If the velocity of sound in the wire string is 5280m/s. How far apart are the two men. (5 marks)

(b) How long will it take a 240V, 3000W electric immersion heater to raise the temperature of 150 litres of water in a well lagged copper tank of mass 20kg from 15℃ to 70℃. Also find the cost at 5 shillings per kwh. (5 marks)

8. (a) Mr. Jacob said that “Stars and planets are all the same since they are all found in the space (sky)”. Argue for or against the statement by giving six (6) points to support your answer. (6 marks) 

(b) The greenhouse effect is the process in which the emission of radiation by the atmosphere warms the earth’s surface. This effect is useful to all life on the earth’s surface, especially in maintaining the moderate temperature on the earth. There are some gases which are associated with this effect, and if these gases are present in high amount, they disturb temperature on the earth’s surface. Explain briefly four of these gases. (4 marks)

SECTION C (25 Marks)

Answer any two (2) questions

9. (a) Imagine that the room in which you are seated is filled with a uniform magnetic field pointing vertically downward. At the entre of the room, an electron is released with a certain speed in the horizontal direction. Describe the motion and path of an electron in the field with the help of Fleming’s left-hand rule. (6 marks) 

(b) The figure 3 below shows a top view of a security door, perhaps in a bank. Study carefully the diagram and explain the mechanism of opening the door when the security officer presses a switch to let a customer in a bank. Also explain the function of the springs.  (6.5 marks)

Figure 3: A door lock

10. A beam of high velocity electrons in a vacuum tube is directed towards a piece of tungsten embedded in a block of copper.

(a) Give an account on energy changes which takes place on impact. (3 marks)

(b) How and why should the apparatus be suitable shielded? (5 marks)

(c) Why it is necessary to use a copper block? (4.5 marks)

(Use a single well labeled diagram to support your answers in a, b and c above) 

11. (a) An alternating potential difference is applied across a semiconducting silicon diode.

(i) Sketch a suitable graph to show how the current passing through the diode in forward bias varies with the applied voltage. (3 marks)

(ii) Explain how two semiconducting diodes may be used to charge car accumulator by using 12V alternating voltage? Show the circuit diagram. (5.5 marks) 

(b) Look and study carefully the two diagrams below. The bulb lights in only one of these circuits. Identify the circuit and explain why the bulb lights while in the other circuit does not. (4 marks)

(i)


(ii)
Figure 4

FORM FOUR PHYSICS EXAM SERIES 16  

FORM FOUR PHYSICS EXAM SERIES 16  

THE UNITED REPUBLIC OF TANZANIA PRESIDENT’S OFFICE REGIONAL ADMINISTRATION AND LOCAL GOVERNMENT

MWANZA CITY JOINT EXAMINATION (NYEGEZI, BUTIMBA AND MKUYUNI WARDS)

FORM FOUR PRE - MOCK EXAMINATION-2021

031 PHYSICS

Time: 3 Hours       MAY, 2021  

Instructions

  1. This paper consists of section A, B and C with total of eleven (11) questions.
  2. Answer all questions in section A and B and any two (2) questions in section C.
  3. Cell phones are not allowed in the examination room.
  4. Non programmable calculator and mathematical table are allowed in the examination room.
  5. Where necessary the following constants may be used:
  • Acceleration due to gravity g = 10 m/s2
  • Specific heat capacity of water = 4200Jkg-1K-1
  • Speed of light in air =
  • Speed of sound in air =
  • Pie (π) = 3.14

SECTION A (15 Marks)

Answer all questions in this section

1. For each of the items (i) – (x), choose the correct answer among the given alternatives and write its letter beside the item number. (10 marks @1 mark)

(i) The diagram below shows series of experiments performed by a student to determine the relative density of a cork that floats in water. Four reading of the spring balance were obtained.

The relative density of the cork is given by;-

A.  B.  C.  D. E.

(ii) Diffusion occurs more quickly in a gas than in a liquid because;-

  1. The liquid contains a layer on its surface
  2. The gas contain semi-permeable membrane
  3. The gas molecules is small in size compared to the liquid molecules
  4. The adhesion is large than cohesion in gas compared to that in liquid.
  5. The speed of molecules in gas is greater than in liquid.

(iii) In a loading a lorry a man lifts boxes each of weight 100N through a height of 1.5m, if he lifts 4boxes per minute, the average power the man is working is;-

  1. 100
  2. 10 
  3. 600
  4. 37.5
  5. 2250

(iv) In a process of charging by induction in static electricity;-

  1. A conductor is rubbed with an insulator
  2. A charge is produced by friction
  3. Negative and positive charges are separated
  4. A positive charge induces a positive charge
  5. Electrons are sprayed into an object

(v) A magnetic needle, free to turn in a vertical plane, is suspended first at the earth’s south magnetic pole and then at a point on the magnetic equator. The respective angles between the needle and horizontal are:

  1. 0o and 0o 
  2. 70o and 70o  
  3. 0o and 45o 
  4. 0o and 90o
  5. 90o and 0o

(vi) The acceleration of a moving object may be found from

  1. The area under its velocity-time graph.
  2. The slope of the velocity-time graph.
  3. The area under its distance-time graph.
  4. The slope of the distance-time graph.
  5. The slope of the peak of its distance-time graph.

(vii) Which of these resources of energy is non-renewable?

  1. Wave energy 
  2. Bio fuels
  3. Radiant energy 
  4. Fossil fuel
  5. Geothermal energy

(viii) A typist uses a new carbon paper under her top typing paper for making a copy of a letter. When she holds the carbon paper close to a plane mirror, she can read the letter. This is because the mirror;

  1. Forms an image the same size as object
  2. Produces an inverted image
  3. Produces a laterally image
  4. Forms a virtual image
  5. Forms an image behind the mirror.

(ix) Light waves differ from sound waves because;

  1. Light is an electromagnetic wave
  2. Light wave are long and sound waves are short
  3. Interference is obtained with light waves but not with sound waves.
  4. The speed of light is independent of the medium in which it travels
  5. Sound waves do not travel in water

(x) The layer in the atmosphere where weather phenomena are formed is;-

  1. Stratosphere 
  2. Magnetosphere 
  3. Troposphere 
  4. Thermosphere
  5. Exosphere

2. Match the items in List A with the responses in List B by writing the letter of the corresponding response beside the item number in the answer sheet provided.

LIST A

LIST B

(i) Longitudinal wave

(ii) Reverberation

(iii) Snell’s law

(iv) Critical angle

(v) Principle of superposition

  1. Sin  over Sin
  2. Refractive index
  3. Water waves
  4. Sound wave
  5. Angle of incidence for which the angle of refractive is 900
  6. Rise and fall of sounds of two vibrating objects
  7. The resultant displacement at any point is equal to the sum of displacements of different waves
  8. The multiple reflection of sound waves
  9. Angle of reflection for which the angle of incidence is 900

 SECTION B (60 Marks)

Answer all questions from this section

3. (a) A body dipped in a liquid experiences an upthrust. Explain three factors on which the upthrust depends. (4.5 marks)

(b) Two identical free running trolleys are on a smooth horizontal runway. One trolley is at rest and the other approaches it at constant speed of 20m/s.

(i) Use the principle of conservation of momentum find the common speed of two trolleys after the collision. (3.5 marks)

(ii) Why the kinetic energies before and after the collision are different? (2 marks)

4. (a) A uniform half metre rule is balanced at 15cm mark when a load of 0.4N is hanging at the zero mark. Draw a sketched diagram indicating the arrow of weight of the rule acting through the centre of gravity hence determine the weight of the half metre rule (5 marks)

(b) A screw jack has a screw pitch of 5mm and the effort arm of 16cm .

(i) State two forms of energy in which the energy supplied to the screw jack is finally converted to.(2 marks)

(ii) Determine the percentage efficiency of this screw jack, if it needs an effort of 30N to lift a load of 750N.(3 marks)

 5. (a) State one example of the use of a convex mirror and indicate why it is preferred to a plane mirror. (4 marks)

(b) An object is set 20cm in front of a lens and the real, inverted, magnified and at great distance image was formed. State the type of the lens used and determine the value of focal length.

(6 marks)

6. (a) Describe how a lens camera operates the same as human eye. Give three points. (6 marks) 

(b) A 42kg refrigerator is placed on the back of a stationary pick-up. The coefficient of static friction between the refrigerator and the pick-up bed is 0.44. At what rate can the pick-up accelerate without the refrigerator sliding off the back? (4 marks)

7. (a) The diagram below show a bimetallic thermostat used to regulate a cooler and heater in a class room. It consist a brass of linear expansivity and iron of linear expansivity . To keep the temperature in the room constant, which of the two devices A or B should be the heater? Explain your answer.

(5 marks)

(b) Three beakers are of identical size and shape; one beaker is painted matt black, one is dull white and one is gloss white. The beakers are filled with boiling water.

(i) In which beaker will the water cool most quickly? Give a reason. (3 marks)

(ii) State a process in addition to conduction, convection and radiation, by which heat energy will be lost from the beaker. (2 marks)

8. (a) A pressure cooker will cook beans faster than an open saucepan. Give explanation on these observations.(5 marks)

(b) An insulated cup holds 0.3kg of water at 0oC. 0.2kg of boiling water at standard pressure is poured into the cup. What will be the final temp (5 marks)

 SECTION C (25 Marks)

Answer two (2) questions from this Section

9. (a) A boy has a large number of coloured bulbs labeled 240V and 60W, he wishes to use for decoration so that the bulbs operates normally. How many can he connect to a 240V and 5A fuse? (4 marks)

(b) You are required to use 3Ω resistor in your electric circuit. Unfortunately there are only 2Ω resistors in School Physics laboratory. How could you design the required resistor from available resistors? (4.5 marks)

(c) A figure shows a block diagram of generation and distribution of electricity using a.c generator. Study the diagram then answer the questions that follow.

  1. What will be the main item of equipment in the sub-stations A and B? (1 mark)
  2. If the transformer T has 80 turns on its primary coil how many turns must it have on its secondary coil? (3 marks)

10. (a) A piano wire and a turning fork produce different notes at the same time and beats are heard. What could be done on the piano wire in order to emit a note of the same frequency as vibrating fork.(Give three points) (3 marks)

(b) A graph below shows a wave emitted by the electromagnetic source. If the velocity of the wave is 12m/s, determine the frequency and the wavelength of the wave.

 

(5 marks)

(c) Boats use “sonar” to detect the depth of water in the seas or lakes. A sonar pulse sent out by a boat arrives back after 3seconds. If the speed of sound in water is 1500m/s, how deep is the water (4.5 marks)

 11. (a) Why some semiconductors called ‘P’ type and other ‘N’ type? (3 marks)

(b) Describe, how you will connect the semiconductor diode as forward bias. (4 marks)

(c) The diagram bellow shows a puzzle box contains two lamps and simple components so that when T1 is connected to the anode lamp L1lights but when terminal T2 is connected to the anode, lamp L2 light. Suggest what the puzzle box is and how the connections are made. (5.5 marks)

FORM FOUR PHYSICS EXAM SERIES 70  

FORM FOUR PHYSICS EXAM SERIES 70  

THE PRESIDENT’S OFFICE REGIONAL ADMINISTRATION AND LOCAL GOVERNMENT 

KILOSA DISTRICT COUNCIL 

MIKUMI - CLUSTER ACADEMIC UNITY (MAU) 

FORM FOUR PRE - MOCK JOINT EXAMINATION

032/1 PHYSICS 1

(For School Candidates Only)

TIME: 3 Hours Wednesday March 17, 2021

INSTRUCTIONS

  1. This paper consists of sections A, B and C with a total of eleven (11) questions
  2. Answer all questions in section A and B and two(2) questions from section C
  3. Whenever calculations are involved show your work Cleary
  4. cellular phones and any authorized materials are NOT allowed in the examination room
  5. Non programmable calculator may be used
  6. Write your examination number on every page for your answer booklet(s)

If necessary the following constants may be used

  1. Acceleration due to gravity, g = 10m/s2
  2. Pie , π = 3.14
  3. Resistivity of steel = 10.5 x 10-8 Ωm
  4. At STP the pressure of oxygen and nitrogen = 1.013 x 105 Pa
  5. At STP temperature of oxygen and nitrogen =0°C = 273K 
  6. Density of water = 1g/cm3 or 1000kg/m3

SECTION A (15 MARKS)

Answer all questions in this section

1. For each of the items (i) – (x), choose the correct answer from among the given alternatives and write its letter beside the item number in the answer sheet provided (10 marks) i. Which among of the following is a dwarf planet?

  1. Mercury 
  2. Saturn 
  3. Neptune 
  4. Pluto 

ii. In the resolution of vectors which mathematical method is used?

  1. Pythagoras theorem 
  2. trigonometric ratio 
  3. set theorem 
  4. logarithm 

iii. When a body sliding on an inclined plane its coefficient of dynamic friction can be given as 

  1. Tan θ 
  2. Sin θ 
  3. Cos θ 
  4. mgcos θ

iv. Film in a lens camera is compered to …………….in human eye

  1. Iris 
  2. Retina 
  3. Lens 
  4. Pupil

v. An instrument used to identify the presence of electric charge on an object is called

  1. Telescope 
  2. barometer 
  3. gold-leaf electroscope 
  4. Vernier caliper 

vi. Decreasing in velocity or slowing down of velocity per unit time is called

  1. Acceleration 
  2. Retardation 
  3. Speed 
  4. velocity 

vii. The SI Unit of average velocity is the same as the SI Unit for…….

  1. Length 
  2. Acceleration 
  3. Speed 
  4. Displacement 

viii. An object has density of 7g/cm3. What is its relative density

  1. 5

ix. A stone of 2Kg falls from a height of 25m above the ground. Calculate the potential energy possessed by the stone

  1. 500J 
  2. 270J 
  3. 520J 
  4. 390J

x. A steel bar has a length of 2.3m and diameter of 2 x 10-5 m. what is resistance?

  1. 768.27 Ω 
  2. 668.72 Ω 
  3. 768 Ω 
  4. 769 Ω

2. Match the regions of atmosphere in LIST A with their corresponding characteristics in LIST B by writing the letter of the correct response beside the item number in the answer sheet provided (5 marks)

List A

List B

i. Troposphere

ii. Stratosphere

iii. Mesosphere 

iv. Thermalsphere

v. Exosphere

  1. The region where satellites obit the earth.
  2. The region with ozone layer
  3. The region where clouds and rain are formed
  4. Allow radio communication over long distances
  5. The region where planes fly
  6. The region with the sun
  7. The region where most meteors burn while entering the earth’s surface

SECTION B (60 MARKS) 

Answer all questions in this section

3. (a).A plane mirror and concave mirror both they produce an image from the given object. Draw the diagram to represent the formation of image in each case (for the concave mirror consider an object to be at the center of curvature, (C). Hence show two similarities and two differences of images formed. ( 6 marks)

(b) Explain the formation of mirage (4 marks)

4. (a) A heavy uniform beam AB of weight 500N is supported at its ends. The beam carries a weight of 3000N at a distance of 1.5m from the end A. if the beam is 4m long. Find the thrust at A and B (6 marks)

(b) Machine is a device used to simplify work. With example explain how the following simple machines do they simplify work in their respective stated area. (4 marks)

  1. Inclined plane at home
  2. Screw jack in garage

5. (a) Explain four (4) factors affecting resistance of a conductor. (6 marks)

(b) Describe two (2) defects of simple cell (4 marks)

6. (a) Explain what happens when water in a container heated? (4 marks)

(b) A sample of oxygen gas has a volume of 0.11 m3 at a temperature of 12°C and a pressure of 8.1 x 105 Pa while a sample of nitrogen gas has a volume of 0.18 m3 at a temperature of 22°C and a pressure of 1.03 x 105 Pa. Which gas will have the larger volume at STP? (6 marks)

7. (a) A diagram below illustrates part of the displacement-time graph of a wave travel with velocity of 2m/s. calculate the amplitude, Frequency and Wave length (6 marks)


(b) With their function identify any four main parts of ripple tank (4 marks)

8. (a) With the help of separate diagram explain how three radiations emitted by radioactive elements are affected in electric plates and magnetic plates (6 marks)

(b) A radioactive element has an initial count rate of 1200 counts per minutes measured by a scale and this falls to 150 counts per minute in 15hours. Determine the half-life of the element (4 marks)

SECTION C (25 MARKS) Answer two (2) questions from this section

9. (a) When a simple pendulum displaced at a small angle swings to and fro, in this motion potential energy and kinetic energy changes by alternating each other. With the aid of diagram verify the alternation of these energies. (8.5 marks)

(b) A 50kg girl runs up a staircase of 50 steps each step is 15cm in height in 5s. Find Work done against gravity by the girl and Power she use to run (4 marks)

10. (a) state laws of electromagnetic induction (4.5 marks)

(b) With reasons name the following symbols. Hence state the function of each in the national grid of electricity (8 marks)

(i)

(i)

11. (a) Draw a well labelled diagram of cathode ray tube. Hence state two uses of the drawn diagram and two properties of rays produced in it. (8.5 marks)

(b) What are the main purpose of connecting a diode and transistor on various electronic devices? (Describe for each device) (4 marks)

FORM FOUR PHYSICS EXAM SERIES 51  

FORM FOUR PHYSICS EXAM SERIES 51  

THE PRESIDENT’S OFFICE MINISTRY OF EDUCATION, REGIONAL ADMINISTRATION AND LOCAL GOVERNMENT

ARUSHA REGIONAL MOCK EXAMINATION-MAY 2021 

PHYSICS

FORM FOUR

INSTRUCTIONS

  1. This paper consists of section A, B and C.
  2. Answer all question in section A and B and ANY TWO (2) question from section C
  3. All work should be shown clearly on each question in systematic manner
  4. Mathematical table and non-programmable calculators may be used
  5. Cellular phones and any unauthorized materials are NOT ALLOWED in the examination room.
  6. Write your examination number at the right corner of every page of answer sheets provided
  7. When ever necessary the following constants may be used.
  • Acceleration due to gravity g = 10m/s2 or ION/ kg
  • Pie, π= 3.14
  • Density of water = lg/cm3 or 1000kg/m3
  • Velocity of sound in air = 340m/ s
  • Velocity of sound in water = 1500m/ s

SECTION A (15 MARKS)

1. Choose the most correct answer and write its letter besides its alternatives number it answer sheet/ booklet(s) provided

(i). . . . . . . . . Is the area around a magnet current carrying conductor where magnetic strength can be detected by compass

  1. Magnetic domain
  2. Magnetic field
  3. Magnetic poles
  4. Induced field
  5. Neutral point

ii. Which of the following apparatus is used for measuring the volume of irregular solid?

  1. Pipette
  2. Beaker
  3. Measuring tape iii.
  4. Measuring cylinder
  5. Burette

iii. A suspended magnetic needle always comes to rest with axis in a vertical plane called

  1. Geographic meridian
  2. Magnetic declination
  3. Magnetic meridian
  4. Geographic North pole
  5. Geographic declination

iv. A body weights 3.6N in air and 2.4N when immersed totally in water. The density of the body in g/cm3 is

  1. 0.40g/cm3
  2. 1.40g/cm3
  3. 3.OOg/cm3
  4. 2.00g/cm3
  5. 0.33g/cm3

v.A dish of liquid is left on a laboratory bench, some of the liquid evaporates, what happens and why?

  1. The liquid cools, because liquid molecules have more potential energy than gas molecules
  2. The liquid cools because faster moving molecules escape
  3. The liquid warms because liquid molecules have less potential enerærr than gas molecules
  4. The liquid warms because slower moving molecules are left behind
  5. None of the above

vi. Which particle is emitted by a hot metal filament?

  1. Alpha particle
  2. Neutrons
  3. Electrons
  4. Protons
  5. Beta particle

vii. When Fleming's left hand is applied to a motor the thumb points in the direction of

  1. Field
  2. Current
  3. Motion
  4. Wire
  5. Fingers

viii.When an electric heater is stamped 240v, 60w is to be used in a 240v mains, the resistance of the heater is:-

  1. 960Ω
  2. 96Ω
  3. 9.60Ω
  4. 0.96Ω
  5. 9600Ω

ix. The best way of testing the strength of a magnet is

  1. Republsion 
  2. Attraction 
  3. Harm 
  4. Induction
  5. Heating

x. When secondary colour is mixed with a primary colour the result is a complementary colour. Which of the following colors is complementary to magenta?

  1. Green 
  2. Red 
  3. Blue 
  4. Cyan 
  5. Yellow

2. Match each of the statement in List B. with appropriate word/prase in List A by writing the letter of the response beside the item number in answer sheets/ booklet(e) provided

LIST A

LIST B

i) Primary Rainbow 

ii) Complementary colour 

iii) Subtractive mixing of colors 

iv) Primary color

v) Addictive mixing of colors

  1. Blue, magenta
  2. Red, cyan
  3. cyan, magenta, yellow
  4. It decreases wave length from total mixture
  5. When two primary colors are mixed the colour are subtracted from white light and the mixture becomes black
  6. When two rim colors combine to form white
  7. It increases the wave length present
  8. Is formed as a results of light that has undergone two total internal reflection in water droplet.
  9.  Is formed as a results of light that has undergone one total internal reflection in water droplet.

SECTION B (60 MARKS)

Answer ALL question in this section

3. a) A cube of wood of side 5.0cm and density 600kgm-3 is placed in water

i. What fraction of the volume of the wood would be immersed in water?

ii. What force must be applied to the cube so that the top surface of the cube is on the same level as the water surface?

b) A passenger ferry boat with vertical sides has a water-line area of 1500m2. When fully loaded with passengers it sinks by 0.6cm. If the average mass of a passenger is 60kg. Calculate the number of passengers on boat.

4. a) Use a well labeled diagram to explain the working principle of a hydraulic press in a simple machine

b) A uniform pencil AB weighing 40g can be balanced horizontally on a knife edge at 2cm from the end A when a mass of 60g is hung from this end. What is the length of a pencil?

5. a) Electrical energy is distributed in all parts of Tanzania by the National grid system  which transmits a.c at very high voltage

  1. explain why a very high voltage is necessary
  2. why is an a.c and not a d.c used?
  3. State the advantage the National Grid has over the power station distributing power in the neighboring area directly.

b) The National Electric company (TANESCO) sells electric energy at Tsh. 435 per unit kilowatt/ hour. Calculate the cost of using, a 3kW electric heater and 1500W shaving machine in a barbar shop simultaneously for 10 minutes.

6. a) i) Describe an experiment to show that sound can not travel in a  vaccum 

ii) Why are echoes not formed in class rooms?

iii) How are unwanted echoes example in concert halls eliminated?

b) A drum at station A is connected to a wire at station B. A man at A beats the drum while another person at B places his ear at the wire and hears two sounds separated by a time interval of 0.5 seconds. How far a part are the two men? If the velocity of sound in string is 5280m/s

7. a) Explain briefly how heat losses have been prevented in a vaccum flask.

b) i) Ice is heated from a temperature of -50C to water at 150C. Sketch a graph to show the variation of its density with temperature.

ii) Describe a Simple experiment you can perform in the laboratory to demonstrate that solids expands when heated.

iii) The figure below shows two bulbs A and B painted black and silver respectively. What would you expect to happen if the heater is switched on, given that the bulbs contain air.  Explain.

8. a) i) Define critical angle

ii) Find the refractive index from air to water if refractive index from water to air is 0.75

b) Carefully study the figure below and answer the following questions

i. Give the name of the figure above ii. Label the parts A, B, C, D, E, and G iii. State the functions or part labeled D, E and B iv. Describe the nature of the image formed.

SECTION C (25 MARKS)

Answer two (2) questions from this section

9.(a) Describe impulse of a force and state its Sl unit

(b) A voleeyba]l A of mass 0.3kg is moving on the ground at a speed of 4ms-1 towards a football B which is at rest on the ground and has a mass 0.2kg. when A collides with B, the balls remain in contact for 0.2sec. after the collision B moves with a velocity of  1.8ms-1 in the opposite direction with A. Find

  1. the velocity of A after the collision 
  2. the average force of A and B during the collision

c) State the relationship between the total kinetic energy of the system before and after the collision

10. a) Distinguish between the following:

i) Hard x-ray and soft x-ray

b) A radon 22280 Rn emits α- particles followed by a β - particles. What are the atomic number and mass number of the nucleus formed after the emission?

c) A radioactive source has a half life of 16 days. How long will it take for the count rate to fall from 160 counts per minutes to 20 counts per minute?

d) Mention layer of the Earth and it importance

11. a) Define the following:

  1. A capacitor
  2. An inductor
  3. Transistor

Fig. 01

b) Use the sketch below (fig 01) to answer the following

  1. What do symbol B, C and E represents?
  2. What types of the device does the sketch represent?
  3. Write down the current relation for the device 
  4. What type of a device does the sketch below (fig. 02) represent?

Fig. 02

v. Give the main difference between a device of fig. 01 and fig. 02

c) Draw a circuit for voltage amplification. 

FORM FOUR PHYSICS EXAM SERIES 24  

FORM FOUR PHYSICS EXAM SERIES 24  

Download Learning
Hub App

For Call,Sms&WhatsApp: 255769929722 / 255754805256